Укажите верное определение для индуктивности катушки: Укажите верное определение для индуктивности катушки

Содержание

Определение — взаимная индуктивность — Большая Энциклопедия Нефти и Газа, статья, страница 1

Определение — взаимная индуктивность

Cтраница 1

Определение взаимной индуктивности двух коаксиальных катушек с одинаковыми диаметрами ( рис. 7 — 11, а) может быть сведено к определению собственных индуктивностей четырех катушек.  [1]

Определение взаимной индуктивности двух концентрических коаксиальных катушек, имеющих прямоугольное сечение и одинаковую длину ( рис. 7 — 10, а), может быть сведено к определению собственных индуктивностей четырех катушек той же длины.  [2]

Для определения взаимной индуктивности двух катушек собрана цепь по схеме рис. 2.67. В первичную ветвь этой цепи для измерения тока / j включен амперметр.  [3]

Для определения взаимной индуктивности М двух катушек, надетых на сердечник из немагнитного материала, были проведены два опыта по схеме фиг.

 [4]

Для определения взаимной индуктивности М двух катушек, надетых на сердечник из немагнитного материала, проведены два опыта по схеме фиг.  [5]

Для определения взаимной индуктивности двух реактивных катушек, соединенных последовательно, были проведены измерения мощности и тока при U 220 в и / 50 гц.  [6]

Для определения взаимной индуктивности двух катушек была собрана цепь по схеме рис. 5.3. В первичную ветвь этой цепи для измерения тока / 1 включен амперметр.  [7]

Для определения взаимной индуктивности следует предположить ток в одном из контуров и определить создаваемое им потокосцепление с другим контуром. Принципиально безразлично, задаваться ли током в линии или в контуре.  [8]

Для определения взаимной индуктивности двух катушек была собрана цепь по схеме рис. 5.30. В первичную ветвь этой цепи для измерения тока 1г включен амперметр.  [9]

Для определения взаимной индуктивности системы, состоящей из тороидальной катушки и прямолинейного провода, вычислим создаваемый проводом магнитный поток Ф0 через поперечное сечение тороида.  [10]

Для определения взаимной индуктивности контуров применяем метод однократного численного интегрирования.  [11]

Так, определение взаимных индуктивностей упомянутых выше массивных колец прямоугольного сечения можно свести к определению собственных индуктивностей нескольких колец такого рода, если в формулах ( 1 — 50) — ( 1 — 58) под F ( & XO принимать произведение skSiM ki, а под F ( k) — произведение s L k, где L k — собственная индуктивность k — ro кольца.  [12]

В некоторых случаях для определения взаимной индуктивности неодинаковых контуров с параллельными осями могут быть использованы приводимые ниже формулы, дающие М в виде бесконечных рядов.  [13]

Уравнение ( 13 — 95) действительно и для определения взаимной индуктивности многовитковых катушек.  [14]

Формула ( 1 — 10) остается справедливой и для определения взаимной индуктивности между двумя замкнутыми витками, у которых размеры поперечного сечения проводов на порядок и более меньше других геометрических размеров.  [15]

Страницы:      1    2

Тест. Явление самоиндукции. Индуктивность

Будьте внимательны! У Вас есть 10 минут на прохождение теста. Система оценивания — 5 балльная. Разбалловка теста — 3,4,5 баллов, в зависимости от сложности вопроса. Порядок заданий и вариантов ответов в тесте случайный. С допущенными ошибками и верными ответами можно будет ознакомиться после прохождения теста. Удачи!

Список вопросов теста

Вопрос 1

На рисунке изображена схема электрической цепи. В какой последовательности зажигаются электрические лампочки при замыкании цепи?

Варианты ответов
  • 2, 1
  • Лампочки зажигаются одновременно
  • 1,2
  • Лампочки не зажгутся
Вопрос 2

Размерность какой из перечисленных величин выражается через основные единицы в СИ как кг · м2/ А2 · с2?
 

Варианты ответов
  • Индуктивность контура
  • Магнитный поток
  • Электромагнитная индукция
  • Ток самоиндукции
Вопрос 3

По катушке индуктивностью L1 = 0,6 Гн течет ток I1 = 15 А, а по катушке с индуктивностью L2 = 15 Гн течет ток I2 = 0,6 А.

Сравните энергии магнитного поля этих катушек.

Варианты ответов
  • E1 = E2
  • E1 = E2 = 0
  • E1 > E2
  • E1
Вопрос 4

По замкнутому проводнику протекает ток силой 2 А. Найдите индуктивность контура проводника, если энергия магнитного составляет 8 мДж. Ответ дайте в мГн и запишите его без единиц измерения.

Вопрос 5

По витку проволоки с индуктивностью 20 мГн протекает ток 3 А. Какова энергия магнитного поля (в мДж) в витке? Ответ запишите без единиц измерения.

Вопрос 6

По соленоиду с индуктивностью 50 мГн протекает ток 4 А. Найдите магнитную энергию соленоида.

Варианты ответов
  • 0,4 Дж
  • 400 мДж
  • 0,04 Дж
  • 50 мДж
Вопрос 7

При увеличении силы тока в катушке вдвое, магнитная энергия катушки увеличилась в 4 раза. Что можно сказать об индуктивности этой катушки?

Варианты ответов
  • Индуктивность равна 2 Гн
  • Индуктивность равна 4 Гн
  • Катушка не обладает индуктивностью
  • Об индуктивности ничего нельзя сказать, т.к. при увеличении силы тока вдвое, энергия магнитного поля катушки возрастает в 4 раза, независимо от индуктивности
Вопрос 8

Явление самоиндукции имеет место 

Варианты ответов
  • место в любых случаях изменения силы тока в цепи, содержащей индуктивность.
  • при изменении самой индуктивности.
  • когда индуктивность не изменяется
  • когда сила тока в цепи остаётся неизменной
Вопрос 9

Индуктивность катушки зависит от
 

Варианты ответов
  • формы
  • размеров
  • числа витков
  • наличия (отсутствия) сердечника
  • силы тока, протекающего по её виткам
Вопрос 10

Физическая величина, введённая  для оценивания способности катушки противодействовать изменению силы тока в ней.

Варианты ответов
  • Индуктивность
  • Коэффициент самоиндукции
  • Электромагнитная индукция
  • Магнитный поток

Глава 23.

Закон электромагнитной индукции

Если в магнитном поле находится замкнутый проводящий контур, не содержащий источников тока, то при изменении магнитного поля в контуре возникает электрический ток. Это явление называется электромагнитной индукцией. Появление тока свидетельствует о возникновении в контуре электрического поля, которое может обеспечить замкнутое движение электрических зарядов или, другими словами, о возникновении ЭДС. Электрическое поле, которое возникает при изменении поля магнитного и работа которого при перемещении зарядов по замкнутому контуру не равна нулю, имеет замкнутые силовые линии и называется вихревым.

Для количественного описания электромагнитной индукции вводится понятие магнитного потока (или потока вектора магнитной индукции) через замкнутый контур. Для плоского контура, расположенного в однородном магнитном поле (а только такие ситуации и могут встретиться школьникам на едином государственном экзамене), магнитный поток определяется как

(23. 1)

где — индукция поля, — площадь контура, — угол между вектором индукции и нормалью (перпендикуляром) к плоскости контура (см. рисунок; перпендикуляр к плоскости контура показан пунктиром). Единицей магнитного потока в международной системе единиц измерений СИ является Вебер (Вб), который определяется как магнитный поток через контур площади 1 м2 однородного магнитного поля с индукцией 1 Тл, перпендикулярной плоскости контура.

Величина ЭДС индукции , возникающая в контуре при изменении магнитного потока через этот контур, равна скорости изменения магнитного потока

(23.2)

Здесь — изменение магнитного потока через контур за малый интервал времени . Важным свойством закона электромагнитной индукции (23.2) является его универсальность по отношению к причинам изменения магнитного потока: магнитный поток через контур может меняться из-за изменения индукции магнитного поля, изменения площади контура или изменения угла между вектором индукции и нормалью, что происходит при вращении контура в поле. Во всех этих случаях по закону (23.2) в контуре будет возникать ЭДС индукции и индукционный ток.

Знак минус в формуле (23.2) «отвечает» за направление тока, возникающего в результате электромагнитной индукции (правило Ленца). Однако понять на языке закона (23.2), к какому направлению индукционного тока приведет этот знак при том или ином изменении магнитного потока через контур, не так-то просто. Но достаточно легко запомнить результат: индукционный ток будет направлен таким образом, что созданное им магнитное поле будет «стремиться» компенсировать то изменение внешнего магнитного поля, которое этот ток и породило. Например, при увеличении потока внешнего магнитного поля через контур в нем возникнет индукционный ток, магнитное поле которого будет направлено противоположно внешнему магнитному полю так, чтобы уменьшить внешнее поле и сохранить, таким образом, первоначальную величину магнитного поля. При уменьшении потока поля через контур поле индукционного тока будет направлено так же, как и внешнее магнитное поле.

Если в контуре с током ток в силу каких-то причин изменяется, то изменяется и магнитный поток через контур того магнитного поля, которое создано самим этим током. Тогда по закону (23.2) в контуре должна возникать ЭДС индукции. Явление возникновения ЭДС индукции в некоторой электрической цепи в результате изменения тока в самой этой цепи называется самоиндукцией. Для нахождения ЭДС самоиндукции в некоторой электрической цепи необходимо вычислить поток магнитного поля, создаваемого этой цепью через нее саму. Такое вычисление представляет собой сложную проблему из-за неоднородности магнитного поля. Однако одно свойство этого потока является очевидным. Поскольку магнитное поле, создаваемого током в цепи, пропорционально величине тока, то и магнитный поток собственного поля через цепь пропорционален току в этой цепи

(23.3)

где — сила тока в цепи, — коэффициент пропорциональности, который характеризует «геометрию» цепи, но не зависит от тока в ней и называется индуктивностью этой цепи. Единицей индуктивности в международной системе единиц СИ является Генри (Гн). 1 Гн определяется как индуктивность такого контура, поток индукции собственного магнитного поля через который равен 1 Вб при силе тока в нем 1 А. С учетом определения индуктивности (23.3) из закона электромагнитной индукции (23.2) получаем для ЭДС самоиндукции

(23.4)

Благодаря явлению самоиндукции ток в любой электрической цепи обладает определенной «инерционностью» и, следовательно, энергией. Действительно, для создания тока в контуре необходимо совершить работу по преодолению ЭДС самоиндукции. Энергия контура с током и равна этой работе. Необходимо запомнить формулу для энергии контура с током

(23.5)

где — индуктивность контура, — сила тока в нем.

Явление электромагнитной индукции широко применяется в технике. На нем основано создание электрического тока в электрических генераторах и электростанциях. Благодаря закону электромагнитной индукции происходит преобразование механических колебаний в электрические в микрофонах. На основе закона электромагнитной индукции работает, в частности, электрическая цепь, которая называется колебательным контуром (см. следующую главу), и которая является основой любой радиопередающей или радиопринимающей техники.

Рассмотрим теперь задачи.

Из перечисленных в задаче 23.1.1 явлений только одно есть следствие закона электромагнитной индукции — появление тока в кольце при проведении сквозь него постоянного магнита (ответ 3). Все остальное — результат магнитного взаимодействия токов.

Как указывалось во введении к настоящей главе, явление электромагнитной индукции лежит в основе работы генератора переменного тока (задача 23.1.2), т.е. прибора, создающего переменный ток, заданной частоты (ответ 2).

Индукция магнитного поля, создаваемого постоянным магнитом, уменьшается с увеличением расстояния до него. Поэтому при приближении магнита к кольцу (задача 23.1.3) поток индукции магнитного поля магнита через кольцо изменяется, и в кольце возникает индукционный ток. Очевидно, это будет происходить при приближении магнита к кольцу и северным, и южным полюсом. А вот направление индукционного тока в этих случаях будет различным. Это связано с тем, что при приближении магнита к кольцу разными полюсами, поле в плоскости кольца в одном случае будет направлено противоположно полю в другом. Поэтому для компенсации этих изменений внешнего поля магнитное поле индукционного тока должно быть в этих случаях направлено по-разному. Поэтому и направления индукционных токов в кольце будут противоположными (ответ 4).

Для возникновения ЭДС индукции в кольце необходимо, чтобы менялся магнитный поток через кольцо. А поскольку магнитная индукция поля магнита зависит от расстояния до него, то в рассматриваемом в задаче 23.1.4 случае поток через кольцо будет меняться, в кольце возникнет индукционный ток (ответ 1).

При вращении рамки 1 (задача 23.1.5) угол между линиями магнитной индукции (а, значит, и вектором индукции) и плоскостью рамки в любой момент времени равен нулю. Следовательно, магнитный поток через рамку 1 не изменяется (см. формулу (23.1)), и индукционный ток в ней не возникает. В рамке 2 индукционный ток возникнет: в положении показанном на рисунке, магнитный поток через нее равен нулю, когда рамка повернется на четверть оборота — будет равен , где — индукция, — площадь рамки. Еще через четверть оборота поток снова будет равен нулю и т.д. Поэтому поток магнитной индукции через рамку 2 изменяется в процессе ее вращения, следовательно, в ней возникает индукционный ток (ответ 2).

В задаче 23.1.6 индукционный ток возникает только в случае 2 (ответ 2). Действительно, в случае 1 рамка при движении остается на одном и том же расстоянии от проводника, и, следовательно, магнитное поле, созданное этим проводником в плоскости рамки, не изменяется. При удалении рамки от проводника магнитная индукция поля проводника в области рамки изменяется, меняется магнитный поток через рамку, и возникает индукционный ток

В законе электромагнитной индукции утверждается, что индукционный ток в кольце будет течь в такие моменты времени, когда изменяется магнитный поток через это кольцо. Поэтому пока магнит покоится около кольца (задача 23.1.7) индукционный ток в кольце течь не будет. Поэтому правильный ответ в этой задаче — 2.

Согласно закону электромагнитной индукции (23.2) ЭДС индукции в рамке определяется скоростью изменения магнитного потока через нее. А поскольку по условию задачи 23.1.8 индукция магнитного поля в области рамки изменяется равномерно, скорость ее изменения постоянна, величина ЭДС индукции не изменяется в процессе проведения опыта (ответ 3).

В задаче 23.1.9 ЭДС индукции, возникающая в рамке во втором случае, вчетверо больше ЭДС индукции, возникающей в первом (ответ 4). Это связано с четырехкратным увеличением площади рамки и, соответственно, магнитного потока через нее во втором случае.

В задаче 23.1.10 во втором случае в два раза увеличивается скорость изменения магнитного потока (индукция поля меняется на ту же величину, но за вдвое меньшее время). Поэтому ЭДС электромагнитной индукции, возникающая в рамке во втором случае, в два раза больше, чем в первом (ответ 1).

При увеличении тока в замкнутом проводнике в два раза (задача 23.2.1), величина индукции магнитного поля возрастет в каждой точке пространства в два раза, не изменившись по направлению. Поэтому ровно в два раза изменится магнитный поток через любую малую площадку и, соответственно, и весь проводник (ответ 1). А вот отношение магнитного потока через проводник к току в этом проводнике, которое и представляет собой индуктивность проводника , при этом не изменится (задача 23.2.2 — ответ 3).

Используя формулу (23.3) находим в задаче 32.2.3 Гн (ответ 4).

Связь между единицами измерений магнитного потока, магнитной индукции и индуктивности (задача 23.2.4) следует из определения индуктивности (23.3): единица магнитного потока (Вб) равна произведению единицы тока (А) на единицу индуктивности (Гн) — ответ 3.

Согласно формуле (23.5) при двукратном увеличении индуктивности катушки и двукратном уменьшении тока в ней (задача 23.2.5) энергия магнитного поля катушки уменьшится в 2 раза (ответ 2).

Когда рамка вращается в однородном магнитном поле, магнитный поток через рамку меняется из-за изменения угла между перпендикуляром к плоскости рамки и вектором индукции магнитного поля. А поскольку и в первом и втором случае в задаче 23.2.6 этот угол меняется по одному и тому же закону (по условию частота вращения рамок одинакова), то ЭДС индукции меняются по одному и тому же закону, и, следовательно, отношение амплитудных значений ЭДС индукции в рамках равно единице (ответ 2).

Магнитное поле, создаваемое проводником с током в области рамки (задача 23.2.7), направлено «от нас» (см. решение задач главы 22). Величина индукции поля провода в области рамки при ее удалении от провода будет уменьшаться. Поэтому индукционный ток в рамке должен создать магнитное поле, направленное внутри рамки «от нас». Используя теперь правило буравчика для нахождения направления магнитной индукции, заключаем, что индукционный ток в рамке будет направлен по часовой стрелке (ответ 1).

При увеличении тока в проводе будет возрастать созданное им магнитное поле и в рамке возникнет индукционный ток (задача 23.2.8). В результате возникнет взаимодействие индукционного тока в рамке и тока в проводнике. Чтобы найти направление этого взаимодействия (притяжение или отталкивание) можно найти направление индукционного тока, а затем по формуле Ампера силу взаимодействия рамки с проводом. Но можно поступить и по-другому, используя правило Ленца. Все индукционные явления должны иметь такое направление, чтобы компенсировать вызывающую их причину. А поскольку причина — увеличение тока в рамке, сила взаимодействия индукционного тока и провода должна стремиться уменьшить магнитный поток поля провода через рамку. А поскольку магнитная индукция поля провода убывает с увеличением расстояния до него, то эта сила будет отталкивать рамку от провода (ответ 2). Если бы ток в проводе убывал, то рамка притягивалась бы к проводу.

Задача 23.2.9 также связана с направлением индукционных явлений и правилом Ленца. При приближении магнита к проводящему кольцу в нем возникнет индукционный ток, причем направление его будет таким, чтобы компенсировать вызывающую его причину. А поскольку эта причина — приближение магнита, кольцо будет отталкиваться от него (ответ 2). Если магнит отодвигать от кольца, то по тем же причинам возникло бы притяжение кольца к магниту.

Задача 23.2.10 — единственная вычислительная задача в этой главе. Для нахождения ЭДС индукции нужно найти изменение магнитного потока через контур . Это можно сделать так. Пусть в некоторый момент времени перемычка находилась в положении, показанном на рисунке, и пусть прошел малый интервал времени . За этот интервал времени перемычка переместится на величину . Это приведет к увеличению площади контура на величину . Поэтому изменение магнитного потока через контур будет равно , а величина ЭДС индукции (ответ 4).

Элементы цепи синусоидального тока. Векторные диаграммы и комплексные соотношения для них. (Лекция N 4)

1. Резистор

Идеальный резистивный элемент не обладает ни индуктивностью, ни емкостью. Если к нему приложить синусоидальное напряжение (см. рис. 1), то ток i через него будет равен

. (1)

Соотношение (1) показывает, что ток имеет ту же начальную фазу, что и напряжение. Таким образом, если на входе двухлучевого осциллографа подать сигналы u и i, то соответствующие им синусоиды на его экране будут проходить (см. рис. 2) через нуль одновременно, т.е. на резисторе напряжение и ток совпадают по фазе.

Из (1) вытекает:

;

.

 

Переходя от синусоидальных функций напряжения и тока к соответствующим им комплексам:

;

,

— разделим первый из них на второй:

или

. (2)

Полученный результат показывает, что отношение двух комплексов есть вещественная константа. Следовательно, соответствующие им векторы напряжения и тока (см. рис. 3) совпадают по направлению.

2. Конденсатор

Идеальный емкостный элемент не обладает ни активным сопротивлением (проводимостью), ни индуктивностью. Если к нему приложить синусоидальное напряжение (см. рис. 4), то ток i через него будет равен

. (3)

Полученный результат показывает, что напряжение на конденсаторе отстает по фазе от тока на /2. Таким образом, если на входы двухлучевого осциллографа подать сигналы u и i, то на его экране будет иметь место картинка, соответствующая рис. 5.

Из (3) вытекает:

;

 

.

 

Введенный параметр называют реактивным емкостным сопротивлением конденсатора. Как и резистивное сопротивление, имеет размерность Ом. Однако в отличие от R данный параметр является функцией частоты, что иллюстрирует рис. 6. Из рис. 6 вытекает, что при конденсатор представляет разрыв для тока, а при .

Переходя от синусоидальных функций напряжения и тока к соответствующим им комплексам:

;

,

— разделим первый из них на второй:

или

. (4)

В последнем соотношении — комплексное сопротивление конденсатора. Умножение на соответствует повороту вектора на угол по часовой стрелке. Следовательно, уравнению (4) соответствует векторная диаграмма, представленная на рис. 7.

3. Катушка индуктивности

Идеальный индуктивный элемент не обладает ни активным сопротивлением, ни емкостью. Пусть протекающий через него ток (см. рис. 8) определяется выражением . Тогда для напряжения на зажимах катушки индуктивности можно записать

. (5)

Полученный результат показывает, что напряжение на катушке индуктивности опережает по фазе ток на /2. Таким образом, если на входы двухлучевого осциллографа подать сигналы u и i, то на его экране (идеальный индуктивный элемент) будет иметь место картинка, соответствующая рис. 9.

Из (5) вытекает:


.

Введенный параметр называют реактивным индуктивным сопротивлением катушки; его размерность – Ом. Как и у емкостного элемента этот параметр является функцией частоты. Однако в данном случае эта зависимость имеет линейный характер, что иллюстрирует рис. 10. Из рис. 10 вытекает, что при катушка индуктивности не оказывает сопротивления протекающему через него току, и при .

Переходя от синусоидальных функций напряжения и тока к соответствующим комплексам:

;

,

разделим первый из них на второй:

или

. (6)

В полученном соотношении — комплексное

сопротивление катушки индуктивности. Умножение на соответствует повороту вектора на угол против часовой стрелки. Следовательно, уравнению (6) соответствует векторная диаграмма, представленная на рис. 11

 

4. Последовательное соединение резистивного и индуктивного элементов

Пусть в ветви на рис. 12 . Тогда

где

, причем пределы изменения .

Уравнению (7) можно поставить в соответствие соотношение

,


которому, в свою очередь, соответствует векторная диаграмма на рис. 13. Векторы на рис. 13 образуют фигуру, называемую треугольником напряжений. Аналогично выражение

графически может быть представлено треугольником сопротивлений (см. рис. 14), который подобен треугольнику напряжений.

 

5. Последовательное соединение резистивного и емкостного элементов

Опуская промежуточные выкладки, с использованием соотношений (2) и (4) для ветви на рис. 15 можно записать

., (8)

где

, причем пределы изменения .


На основании уравнения (7) могут быть построены треугольники напряжений (см. рис. 16) и сопротивлений (см. рис. 17), которые являются подобными.

6. Параллельное соединение резистивного и емкостного элементов

Для цепи на рис. 18 имеют место соотношения:

;

, где [См] – активная проводимость;

, где [См] – реактивная проводимость конденсатора.

Векторная диаграмма токов для данной цепи, называемая треугольником токов, приведена на рис. 19. Ей соответствует уравнение в комплексной форме

,

где ;

— комплексная проводимость;

.

Треугольник проводимостей, подобный треугольнику токов, приведен на рис. 20.

Для комплексного сопротивления цепи на рис. 18 можно записать

.

Необходимо отметить, что полученный результат аналогичен известному из курса физики выражению для эквивалентного сопротивления двух параллельно соединенных резисторов.

7. Параллельное соединение резистивного и индуктивного элементов

Для цепи на рис. 21 можно записать

;

, где [См] – активная проводимость;

, где [См] – реактивная проводимость катушки индуктивности.

Векторной диаграмме токов (рис. 22) для данной цепи соответствует уравнение в комплексной форме

,

где ;

— комплексная проводимость;

.

Треугольник проводимостей, подобный треугольнику токов, приведен на рис. 23.


Выражение комплексного сопротивления цепи на рис. 21 имеет вид:

.

Литература

1.     Основы теории цепей: Учеб. для вузов /Г.В.Зевеке, П.А.Ионкин, А.В.Нетушил, С.В.Страхов. –5-е изд., перераб. –М.: Энергоатомиздат, 1989. -528с.

2.     Бессонов Л.А. Теоретические основы электротехники: Электрические цепи. Учеб. для студентов электротехнических, энергетических и приборостроительных специальностей вузов. –7-е изд., перераб. и доп. –М.: Высш. шк., 1978. –528с.

Контрольные вопросы и задачи

1.     В чем сущность реактивных сопротивлений?

2.     Какой из элементов: резистор, катушку индуктивности или конденсатор – можно использовать в качестве шунта для наблюдения за формой тока?

3.     Почему катушки индуктивности и конденсаторы не используются в цепях постоянного тока?

4.     В ветви на рис. 12 . Определить комплексное сопротивление ветви, если частота тока .
Ответ: .

5.     В ветви на рис. 15 . Определить комплексное сопротивление ветви, если частота тока .
Ответ: .

6.     В цепи на рис. 18 . Определить комплексные проводимость и сопротивление цепи для .
Ответ: ; .

7.     Протекающий через катушку индуктивности ток изменяется по закону А. Определить комплекс действующего значения напряжения на катушке.
Ответ: .

Магнитное поле катушки с током — урок. Физика, 8 класс.

Практический интерес представляет собой магнитное поле катушки с током.

Катушка получится, если плотно, виток к витку, намотать провод в достаточно длинную спираль (рис. 1). В катушке может быть несколько десятков, сотен или даже тысяч витков.

Соленоид (от греч. solen — «канал», «труба» и eidos — «подобный») — разновидность катушки с током. Обычно под термином «соленоид» подразумевается цилиндрическая обмотка из провода, причём длина такой обмотки многократно превышает её диаметр.

 

Рис. 1

 

Рассмотрим рисунок 2. Мы видим цепь, состоящую из источника тока, реостата и катушки. Катушка содержит большое число витков провода. При протекании тока по цепи железные опилки притягиваются к торцу катушки. А если тока нет, то притяжение не наблюдается.

 

Рис. 2

 

Если катушка в этом опыте будет подвешена на проводах, то при протекании тока в цепи, она установится в пространстве строго определённым образом. Точно так же, как и магнитная стрелка компаса (в направлении север — юг).

Это наблюдение позволяет сделать вывод, что катушка с током тоже имеет магнитные полюсы (рис. 3).

 

Рис. 3

 

Логично предположить, что у катушки магнитное поле тоже имеется. Для доказательства можно воспользоваться железными опилками  (рис. 4).

 

Рис. 4

 

Железные опилки располагаются, образуя замкнутые кривые.

За направление линий магнитного поля принято направление от северного полюса катушки к южному (вне катушки с током).

 

Сила магнитного поля постоянного магнита невелика. Другое дело – электромагнит. Сила магнитного поля электромагнита можно изменяться. Ее можно увеличивать или уменьшать. Основная часть любого электромагнита – катушка с намотанным на нее проводом. Рассмотрим опыт, изображенный на рисунке 2. По виткам катушки протекает ток, и она притягивает к себе железные предметы (так проявляется магнитное действие тока). Если увеличить количество витков в катушке, не меняя силу тока в ней, то ее магнитное действие усилится, о чем свидетельствует увеличение количества притягиваемых предметов.

 

Магнитное действие катушки с током прямо пропорциональна числу витков в ней.

Соберём электрическую цепь из катушки, реостата (рис. 5), при помощи которого будем изменять силу тока в катушке.

Действие магнитного поля катушки с током прямо пропорционально силе тока.

 

Рис. 5

 

Усиление магнитного поля произойдёт при использовании железного сердечника (рис. 6).

Сердечник — металлический стержень для усиления мощности электромагнита.

Сердечник, введённый внутрь катушки с током, усиливает магнитное действие катушки.

 

Рис. 6

 

Направление магнитного поля тока связано с направлением тока в катушке.

Определить направление линий магнитного поля катушки с током можно при помощи правила правой руки, или правила правого буравчика.

 

Принято считать, что та сторона катушки или витка с током, откуда линии магнитного поля выходят, — это и есть северный магнитный полюс (\(N\)), а сторона, куда линии входят, — это южный магнитный полюс (\(S\)) (рис. 7).

 

Рис. 7

Решение тестов Томский политехнический университет (ТПУ) онлайн

Вопросы тестирования по предмету  — Теоретические основы электротехники.  ТПУ

• Укажите на каком элементе фазовый сдвиг равен минус 90 градусов: 

• Укажите соответствие между обозначением и определением величины для переменного тока, заданного формулой: 

• Укажите правильную формулу для вычисления эквивалентного сопротивления представленной цепи: 

• Установите соответствие между схемами и векторными диаграммами: 

• Определите активную мощность для цепи с параметрами 

• Методом контурных токов 

• Степень магнитной связи индуктивно связанных катушек характеризует: 

• При последовательном соединении индуктивно связанных катушек величина эквивалентного полного сопротивления цепи при встречном включении меньше, чем при согласном включении. 

• Верно ли утверждение, что при идеальном резонансе токов входной ток равен нулю. 

• Укажите правильную формулу для ЭДС симметричного генератора соединенного звездой с нулевым проводом, если напряжение источника: 

• Определите модуль тока если известно сопротивление Z=9.{f}, В 

• Что покажет вольтметр в представленной схеме (вводить цифру) 

• Верно ли утверждение, что при симметричной системе фазных ЭДС (независимо от нагрузки фаз) расчет токов и напряжений можно вести для одной фазы, а затем с помощью фазового множителя найти все остальные токи и напряжения цепи. 

• Верно ли утверждение, что если симметричная система трехфазный ЭДС питает симметричную нагрузку, то ток в нулевом проводе равен нулю. 

• Выберите правильное соотношение для действующих значений фазных (ф) или линейных (л) напряжений и токов трехфазной цепи в симметричном режиме: 

• Почему обрыв нейтрального провода в четырехпроводной системе является аварийным режимом? 

• Как изменятся токи и напряжения цепи, если произойдет обрыв фазы А? Выберете правильный ответ. 

• Как изменятся токи цепи при размыкании выключателя? Указать верный ответ. 

• Измерение мощности осуществляется: 

• В симметричном режиме трехфазной цепи  последовательность чередования фаз. 

• Как изменяться токи цепи после замыкания ключа? Укажите правильный ответ. 

• Выберите верное суждение о несимметричном режиме трехфазной цепи: 

• Укажите правильную формулу для вычисления мощности ваттметра, в представленной схеме: 

 • Указать номер правильной формулы, для определения мощности, регистрируемой ваттметром 

• Как изменится накал ламп групп а, б, в, если произойдет обрыв линии А? Укажите правильный ответ. 

• Возможно ли смещение нейтральной точки приемника n на диаграмме напряжений при включенном нейтральном проводе, если его сопротивление равно нулю? 

• Верно ли утверждение, что ток нейтрального провода с нулевым сопротивлением в несимметричной трехфазной цепи не выравнивает фазные напряжения. 

• Укажите верное количество ваттметров, необходимое для измерения суммарной активной мощности трехфазной цепи без нулевого провода: 

• Укажите формулу для расчета комплекса входного сопротивления четырехполюсника. 

• Укажите вариант ответа, при котором Z1 соответствуют коэффициентам А-формы: A11=1.5, A12=j20 Ом, A22=2. 

• Укажите величину (вводить цифру) сопротивления индуктивного элемента для первой гармоники, если для третьей гармоники — 

• Верно ли утверждение, что действующее значение несинусоидальной величины зависит от частоты гармонической составляющей и не зависит от фазы. 

• Не гармонические периодические сигналы — это токи, напряжения или ЭДС, повторяющиеся через равные промежутки времени, форма которых может быть описана: 

• Укажите величину (вводить цифру) сопротивления емкостного элемента для пятой гармоники, если для первой гармоники — XC(1)=50 

• Укажите правильную формулу расчета действующего значения представленной величины: 

• Как выбирать контуры, чтобы уравнения Кирхгофа для них оказались взаимно независимыми? 

• На рисунке дана структурная схема некоторой цепи (ветви изображены линиями, узлы – точками). Определить для нее число взаимно независимых уравнений, которые можно составить по первому закону Кирхгофа. 

• На рисунке дана структурная схема некоторой цепи (ветви изображены линиями, узлы – точками). Определить для нее число взаимно независимых уравнений, которые можно составить по второму закону Кирхгофа. 

• В чем сущность принципа наложения? Как его проверить на примере цепи показанной на рисунке? В ответе описать необходимые опыты (количество опытов, положения ключей [1] и[2], что нужно сделать с показаниями приборов). Привести расчетные формулы в общем (буквенном) виде, полученные по методу наложения для данной схемы. 

• Поясните принцип взаимности применительно к цепи, показанной на рисунке, и выведите формулы для аналитической его проверки (доказать тождество выражений для двух токов). 

• Возможно ли смещение нейтральной точки приемника на диаграмме напряжений при включенном нейтральном проводе, если его сопротивление равно нулю? 

• Какая нагрузка называется симметричной? 

• Какая нагрузка будет несимметричной? 

• Каковы соотношения между фазными напряжениями симметричного приёмника, соединённого звездой и линейными напряжениями сети? 

• Каковы соотношения между фазными токами симметричного приёмника, соединённого звездой и линейными токами сети? 

• Укажите правильное уравнение, записанное по второму закону Кирхгофа, для предложенной схемы: 

• Почему обрыв нейтрального провода в четырехпроводной системе является аварийным режимом? 

• Верно ли утверждение, что если симметричная система трехфазных ЭДС питает симметричную нагрузку, то расчет токов и напряжений можно вести для одной фазы, а затем с помощью фазового множителя найти все остальные токи и напряжения цепи. 

• Верно ли утверждение, что при расчете цепей с негармоническими сигналами применяется метод наложения, согласно которому расчет схемы проводится для каждой гармоники отдельно, а затем результат записывают в виде суммы рассчитанных гармоник тока или напряжения. 

• Внутреннее сопротивление генератора (в методе эквивалентного генератора) равно эквивалентному сопротивлению цепи относительно зажимов ветви, в которой ищем ток. 

 • Возможно ли смещение нейтральной точки приемника на диаграмме напряжений в трехфазной четырехпроводной цепи при включенном нейтральном проводе, если его сопротивление равно нулю? 

• Укажите на каком элементе фазовый сдвиг равен минус 90 градусов: 

• Укажите условия, соответствующие резонансу: 

• Укажите правильное уравнение, записанное по первому закону Кирхгофа, для предложенной схемы: 

• Определить принужденную составляющую для тока i1(t) (в амперах), если E=100 В, L=1 Гн, R1=R3=100 Ом, R2=25 Ом: 

• Второй закон коммутации — ток через емкость до коммутации iC(0-) равен току через емкость после коммутации iC(0+) 

• Схема, в которой переходного процесса не будет 

• Первый обобщённый закон коммутации 

• Укажите не менее двух вариантов ответа: 

• Уравнение, для определения корней характеристического уравнения (входное сопротивление пассивного двухполюсника) для схемы

 • Укажите последовательность, в которой возрастает величина принуждённой составляющей тока индуктивности iLпр, если E=10 В, R=10 Ом, J=2 А: 

• Укажите номер верного дифференциального уравнения для тока iL(t) переходного процесса исходной схемы: 

• Укажите последовательность, в которой возрастает постоянная времени в схемах: 

• Укажите не менее двух вариантов ответа: 

• При критическом переходном процессе: • Укажите последовательность, в которой возрастает величина тока индуктивности iL(0-) до коммутации, если E=10 В, R=10 Ом, J=2 А: 

• Какие величины выбираются в качестве переменных состояния? 

• Переходная характеристика h(t) зависит от: 

• Как называется реакция цепи в виде тока или напряжения на единичный возмущающий импульс источника при нулевых начальных условиях? 

• Укажите последовательность действий для определения токов и напряжений методом интеграла Дюамеля: 

• Сущность комбинированного операторно-классического метода – применение принципа наложения. 

• Операторная ….. 

• Укажите последовательность действий для определения токов и напряжений операторным методом: • Определите оригинал по известному изображению функции 

• В комбинированном операторно-классическом методе расчета переходных процессов в схеме после коммутации для свободных составляющих индуктивность изображается 

• Выберете все верные формулы для расчета параметров однородной линии 

• Указать номер верного значения коэффициента отражения для напряжения KU  при заданном сопротивлении нагрузки ZH = j100 Ом  и известном волновом сопротивлении линии ZB = 100 Ом: 

 • Уравнение однородной линии в режиме согласованной нагрузки для комплекса напряжения или тока при отсчете координаты х от конца линии: 

• Постоянная уравнения однородной линии для комплекса напряжения • на расстоянии x от конца линии: 

• Короткозамкнутая воздушная линия без потерь длиной 3λ /8 питается от источника напряжением U1 = 100 B, а ее волновое сопротивление 100 Ом. 

• Для определения тока и напряжения в нагрузке линии при расчете переходных процессов 

• Коэффициенты преломления определяются по формулам: 

• Кабельная линия без потерь, нагруженная на емкостное сопротивление, численно равное удвоенному волновому, работает на частоте f = 

• Определите параметры линейной схемы замещения Rd=Rдиф и J0 нелинейного элемента для участка цепи ab его вольтамперной характеристики (ВАХ). 

 • Определить дифференциальное сопротивление Rдиф нелинейного элемента с вольт-амперной характеристикой ,A при U =1 В. 

• Для схемы с параметрами E=50 B, R=25 Ом и заданной вольт-амперной характеристикой нелинейного элемента определить значение тока IR: 

• Определите входное напряжение для схемы с нелинейными элементами (HЭ1) и (HЭ2), ВАХ которых заданы, если показание вольтметра U = 40 B. 

 • Определение закона полного тока: Линейный интеграл от 

 • Статическая индуктивность катушки с током неразветвленной магнитной цепи равна отношению 

• В магнитной цепи непрерывны: 

• Укажите последовательность действий для расчета переходных процессов в нелинейных цепях методом условной линеаризации 

• Определить ток в первый момент после размыкания ключа при E = 10 B и R = 10 

• Выберете верно записанное дифференциальное уравнение для приведенной схемы. 

• Уравнение для метода последовательных интервалов: 

• Цепь первого порядка содержит в после коммутационной цепи только: 

• Выберите верное определение: • независимые начальные условия определяются из расчета схемы после коммутации  -зависимые начальные условия определяются из расчета схемы до коммутации — зависимые начальные условия определяются из схемы после коммутации в установившемся 

• Определить независимые начальные условия (ННУ), если J=3 А, R=50 Ом, L=0.5 Гн: 

• Что такое постоянная времени в цепи первого порядка и как ее определить графически по экспериментальным кривым тока (напряжения)? 

• Переходные токи и напряжения можно представить в виде суммы двух составляющих: затухающей свободной составляющей и незатухающей принуждённой составляющей. 

• Свободные составляющие изменяются экспоненциально, согласно закону 

• Физический смысл постоянной времени следующий: за промежуток времени, равный постоянной времени свободная составляющая уменьшается в e раз. 

• В зависимости от вида корней характеристического уравнения соотнесите тип переходного процесса: 

• Соотнесите величины и формулы с их названиями, используя осциллограмму: 

• Установите соответствие между обозначением и определением параметров свободной составляющей колебательного режима 

• Статические характеристики — это характеристики,… 

• У безынерционных нелинейных резистивных элементов… 

• Какие значения показывают приборы магнитоэлектрической и электродинамической систем?

Решаем и отвечаем на тестовые вопросы за Вас.

Изменится ли активное сопротивление проводника при увеличении

Реактивное сопротивление катушки зависит от частоты тока и индуктивности катушки Реактивное сопротивление конденсатора зависит от частоты тока и емкости конденсатора При уменьшении частоты переменного тока индуктивное сопротивление уменьшается При уменьшении частоты переменного тока активное сопротивление не изменяется

Укажите формулу для определения импеданса цепи переменного тока, состоящей из последовательно соединенных конденсатора и катушки индуктивности:

Укажите единицу СИ емкостного сопротивления: Ом Укажите единицу СИ индуктивного сопротивления: Ом Емкостное сопротивление определяется по формуле:

Укажите формулу для определения импеданса цепи переменного тока, состоящей из последовательно соединенных катушки индуктивности и резистора:

Вектор амплитуды напряжения на резисторе в векторной диаграмме напряжений цепи переменного тока направлен параллельно к оси тока

Вектор амплитуды напряжения на конденсаторе в векторной диаграмме напряжений цепи переменного тока направлен перпендикулярно вниз (под углом — π/2) оси тока

Полное сопротивление — импеданс цепи переменного тока включает активное и реактивное сопротивление

Если угол сдвига фаз между током и напряжением в цепи переменного тока имеет положительное значение, то цепь обязательно содержит катушку индуктивности

Активное сопротивление не зависит от частоты переменного тока

При увеличении частоты переменного тока активное сопротивление не изменяется

Сдвиг фаз между током и напряжением в цепи переменного тока, содержащей катушку индуктивности, резистор и конденсатор определяется по формуле:

Укажите формулу для определения импеданса цепи переменного тока, состоящей из последовательно соединенных резистора, катушки индуктивности и конденсатора:

При прохождении переменного тока в цепи с реактивным сопротивлением происходит

ПРАВИЛЬНЫЕ:

Активное сопротивление цепи не зависит от частоты переменного тока

На векторной диаграмме напряжений в цепи переменного тока вектор амплитуды напряжения на резисторе совпадает по направлению с осью тока

На векторной диаграмме напряжений цепи переменного тока вектор амплитуды напряжения на конденсаторе направлен перпендикулярно оси тока

При увеличении частоты переменного тока индуктивное сопротивление увеличивается (да, зависимость прямопропорциональная)

Угол сдвига фаз между током и напряжением в цепи переменного тока, содержащий конденсатор, имеет отрицательное значение

При прохождении переменного тока на активном сопротивлении происходит выделение теплоты

Переменный ток в цепи с конденсатором опережает напряжение по фазе на π/2

Реактивное сопротивление цепи переменного тока обусловлено наличием в ней конденсаторов и катушек индуктивности

Единицей СИ индуктивного сопротивления является Ом

Переменный ток — ток, изменяющийся во времени

Импеданс — полное сопротивление цепи переменного тока

Емкостное сопротивление уменьшается с увеличением частоты переменного тока

При увеличении частоты переменного тока индуктивное сопротивление увеличивается

Угол сдвига фаз между током и напряжением в цепи переменного тока, содержащий конденсатор, имеет отрицательное значение

НЕПРАВИЛЬНЫЕ:

В цепи переменного тока всегда происходит сдвиг фаз между силой тока и напряжением (не всегда, например, когда индуктивное и емкостное сопротивление равны, не происходит)

Величина (модуль) реактивного сопротивления равна сумме сопротивлений конденсатора и резистора (равна разности)

Ток в цепи переменного тока, содержащей катушку индуктивности, по фазе совпадает с напряжением

При прохождении переменного тока в реактивном сопротивлении происходит выделение теплоты (нет, не происходит, теплота выделяется в активном)

Единицей СИ индуктивного сопротивления является фарад (Ф) (Ом на самом деле, Ф – единица емкости)

Последнее изменение этой страницы: 2016-04-20; Нарушение авторского права страницы

Активное сопротивление — проводник

Активное сопротивление проводника зависит от его размеров и формы. [1]

Активное сопротивление проводника для токов высокой частоты определяется удельным сопротивлением его поверхностного слоя. Для уменьшения этого сопротивления поверхность проводников, работающих в цепях высокой частоты, часто покрывают слоем серебра. [2]

Активное сопротивление проводника зависит от его размеров и формы. [3]

Активное сопротивление проводника различно при постоянном и переменном токе из-за поверхностного эффекта и эффекта близости. [4]

Активное сопротивление проводника увеличивается с увеличением частоты протекающего по нему переменного тока. Для частоты 50 гц, применяющейся в промышленной электротехнике, это увеличение незначительно, а для частот более высоких активное сопротивление проводника может весьма заметно превышать омическое. Кроме того, неравномерное распределение тока обусловливает уменьшение магнитного потока внутри самого провода, вследствие чего уменьшается его индуктивность тем резче, чем выше частота переменного тока, передаваемого по проводу. [6]

Активное сопротивление проводника увеличивается по мере возрастания частоты переменного тока. Например, стальной провод диаметром 5 мм и длиной 1 км имеет при постоянном токе сопротивление, равное 20 омам, а при переменном токе с частотой 20 000 герц — 75 омам. Для частоты переменного тока в 50 герц, применяемого обычно в электрических установках, увеличение сопротивления незначительно. [7]

Активное сопротивление проводников тока определяют измерительными мостами или методом амперметра-вольтметра. Когда нужно знать действительное значение сопротивления проводников, пользуются мостами постоянного тока или методом амперметра-вольтметра. Чтобы установить, находится ли измеряемое сопротивление в допустимых пределах, используют одинарные или двойные автоматические мосты. Для определения процента отклонения измеряемого сопротивления от номинального его значения применяют одинарные или двойные процентные измерительные мосты. [8]

Активное сопротивление проводников токопроводящих частей измеряют методом сопротивления. [9]

Зная активное сопротивление проводника , нетрудно определить и мощность, которая в нем выделяется. [10]

Поскольку активное сопротивление проводника зависит от величины поперечного сечения того поверхностного слоя проводника, в котором течет ток, то для токов СВЧ активное сопротивление тонкостенной трубки не отличается от активного сопротивления проводника в виде сплошного стержня, сделанного из того же металла и имеющего тот же диаметр, что и сплошная трубка. Для уменьшения сопротивления поверхность проводников, применяемых в цепях высокой частоты, часто покрывают слоем металла, обладающего малым удельным сопротивлением, например серебра. [11]

Поскольку активное сопротивление проводника зависит от величины поперечного сечения того поверхностного слоя проводника, в котором течет ток, то для токов сверхвысокой частоты активное сопротивление тонкостенной трубки не отличается от активного сопротивления проводника в виде сплошного стержня, сделанного из того же металла и имеющего тот же диаметр, что и сплошная трубка. [12]

Увеличение активного сопротивления проводников тока якоря и катушек полюсов происходит из-за надрыва и трещин в проводниках или повреждения контактных соединений — распайки концов обмотки в петушках коллектора якоря, ослабления крепления или распайки наконечников. [13]

Под активным сопротивлением проводника понимают такое сопротивление, в котором энергия выделяется в виде теплоты. Электрическая цепь обладает активным сопротивлением К, индуктивностью L и емкостью С, которые являются ее параметрами. [14]

Яэ — активное сопротивление проводника при температуре 9; с8 — удельная теплоемкость проводника при температуре 9; G — масса проводника; а — температурный коэффициент изменения удельного сопротивления; j — сечение проводника; / — длина проводника; с0 — удельная теплоем — кость материала проводника; Р — температурный коэффициент изменения удельной теплоемкости; X — плотность материала проводника; 9Н — начальная температура проводника до КЗ; 9КН — конечная температура проводника во время КЗ; Акн — значение интеграла при верхнем пределе; Аи — значение интеграла при нижнем пределе. [15]

Разделы: Физика

Проведению общественного смотра знаний предшествовало серьезная подготовка.
Учащиеся 11”Б” класса предложили провести открытое мероприятие в виде игры “Слабое звено”, выбрали ведущую, “счетчиков” баллов.
Каждое звено получило за 10 дней игры задание: составить 15–20 вопросов к одной из шести глав учебника “Физика 11”.
Я выбрала из огромного числа 86 вопросов, скорректировала ответы. С ведущей подобрали оптимальный режим озвучивания вопросов, чтобы игроки успели дать блиц-ответы на максимальное число вопросов.
Ребята показали хорошие знания, им не хотелось быть самым слабым звеном в классе, они не желали услышать в свой адрес “колючие” замечания ведущей.
Урок достиг своей цели.
Большинство ребят этого класса поступили в технические вузы на бюджетной основе. Победитель игры – самое “сильное звено” класса, Смирнов Андрей, ныне учится в МФТИ.

Литература:

1) Приложение ПС “Физика”, 1998–2006 год.
2) Учебник “Физика 11” под редакцией А.А. Пинского, М. Просвещение, 1995 г.
3) “Внеклассная работа по физике ” И. Я. Ланина М. Просвещение, 1977 г.

Цель: обобщить полученные знания, повысить творческую активность учащихся, расширить кругозор, развивать умение учащихся кратко выражать мысли, развивать внимание, память, умение сопереживать одноклассникам и адекватно реагировать на поражение и успех.

Оборудование: карточки с вопросами и ответами у «счетчиков» баллов и ведущей.

Учитель: Ребята, мы закончили изучение основного курса физики, и я предлагаю провести урок по проверке знаний в форме игры «Слабое звено». Напоминаю правила игры. Игра состоит из 6 раундов, длительностью 8–10 минут каждый. Конкретный и краткий ответ на вопрос должен быть озвучен в течение 10 секунд. Участники, давшие минимальное количество верных ответов, выбывают из игры. Представляю вам экспертную команду «счетчиков» баллов и ведущую. Приступаем к игре и желаем, чтобы никто из вас не оказался слабым звеном.

1-й раунд

Глава 1. Электромагнитные колебания и основы электротехники

Ведущая:

1. Из чего состоит колебательный контур? (Конденсатор и катушка.)
2. Какие колебания называются гармоническими? (Изменение физических величин с течением по синусоидальному закону.)
3. Что происходит с энергией в колебательном контуре? (Перераспределение энергии.)
4. Назовите формулу циклической частоты. ()
5. Назовите формулу Томсона. ()
6. Назовите Формулу циклической частоты для пружинного маятника. ()
7. Назовите формулу циклической частоты для математического маятника. ()
8. Назовите формулу магнитного потока. ()
9. Назовите условие резонанса. (Совпадение частоты переменного тока с частотой свободных колебаний.)
10. Скажите определение активного сопротивления. (Сопротивление элемента электрической цепи в котором происходит превращение электрической. энергии во внутреннюю.)
11. Изменится ли активное сопротивление проводника при увеличении частоты переменного тока с 20–40 Гц? (Нет.)
12. Кем изобретен трансформатор? (Яблочковым.)
13. Когда был изобретен трансформатор? (В XIX веке.)
14. В каких 2-х режимах работает трансформатор? (Холостой ход и режим нагрузки.)
15. Как обозначается и в чем измеряется индуктивность? (Генри)
16. Какой буквой обозначается добротность? (Q)
17. Назовите формулу закона электромагнитной индукции? ()
18. Назовите единицу измерения мощности? (Ватт.)
19. Назовите формулу мощности в цепи переменного тока. ()
20. Назовите электростанции Татарстана. (Нижнекамская ГЭС, Заинская ГРЭС, Челнинская.)
21. Чему равно полное сопротивление в цепи переменного тока? ()
22. Как найти ? (; )

Ведущая:

– Кто плохо соображает?
– Кто напрасно надеется, что остальные скажут за него все?

(Из игры выбывает учащиеся с меньшим числом верных ответов)

2-й раунд

Глава 2. Электромагнитные волны и основы радиопередачи

1. Назовите свойства радиоволн. (Интерференция, поляризация, дифракция.)
2. Кто изобрел радио? (Попов.)
3. Что явилось основной деталью радиоприемника Попова? (Когерер.)
4. Какое явление называют модуляцией? (Процесс, с помощью которого амплитуда, частота или фаза медленно изменяются.)
5. Что такое детектирование? (Преобразование модулированных колебаний в систему кратковременных импульсов.)
6. В каком устройстве возникают незатухающие электромагнитные волны? (Генератор незатухающих колебаний на транзисторе.)
7. Кто в 1864 году высказал гипотезу о существовании электромагнитных волн, способных распространятся в вакууме? (Максвелл.)
8. Кем в 1887 году экспериментально были обнаружены электромагнитные волны? (Герцем.)
9. Назовите формулу поверхностной плотности потока излучения? ()
10. При отражении каких волн мы слышим эхо? (Звуковых.)
11. Как называется явление изменения направления распространения волн на границе двух сред? (Преломление.)
12. Что называется интерференцией? (Явление увеличения или уменьшения амплитуды результирующей волны при сложении двух или нескольких волн с одинаковыми частотами колебаний.)
13. Выполняется ли закон сохранения энергии при интерференции? (Да.)
14. Как называется отклонение направления распространения света от прямолинейного у края преграды? (Дифракция.)

Ведущая:

– Кто не стоит ломаного гроша?
– Кто мчится к пропасти на всех парах?

(Из игры выбывает учащиеся с меньшим числом верных ответов)

3-й раунд

Глава 3. Световые волны

1. Когда и кем измерена скорость света? (Рёмер; в конце XVII века.)
2. Чему равна скорость света? ()
3. Назовите немецкого физика, лауреата Нобелевской премии, открывшего лучи, которые носят его имя? (Рентген.)
4. Назовите явление, присущее световым волнам? (Интерференция, преломление, дифракция, поляризация, отражение.)
5. Какие волны называют когерентными? (Световые волны одинаковой частоты, у которых разность фаз равна нулю.)
6. Какое явление называется дифракцией? (Явление огибания светом контуров непрозрачных предметов и проникновения света в область геометрической тени.)
7. Какое явление называется дисперсией, и кем оно было открыто? ( Это зависимость скорости света от частоты волны; Ньютон.)
8. Монохромотичен или нет белый свет? (Нет.)
9. Является ли интерференция света доказательством, что свет обладает волновыми свойствами? (Да.)
10. Как называются две волны, фазы которых меняются случайно и независимо друг от друга? (Некогерентные.)

Ведущая:

– Кто сломал зубы об гранит науки?
– Кто завышает уровень своего IQ?

(Из игры выбывает учащиеся с меньшим числом верных ответов)

4-й раунд

Глава 4. Оптические приборы

1. Назовите закон преломления света? ()
2. В каком разделе оптики изучается формирование изображений при распространении света по световодам? ( Волоконная оптика.)
3. Как называется изображение, которое получается за счет пересечения не самих лучей, а их продолжений? (Мнимое.)
4. Назовите прозрачное стеклянное тело ограниченное двумя сферическими поверхностями с радиусами кривизны и ? (Линза)
5. Как называется прямая, на которой лежат центры обеих сферических поверхностей линзы? (Главная оптическая ось.)
6. Единица измерения оптической силы линзы? (Диоптрия)
7. Назовите формулу тонкой линзы? ()
8. С помощью, каких рецепторных клеток глаза осуществляется цветное зрение: колбочек или палочек? (Колбочек.)
9. Как называется дефект глаза, при котором ближняя точка удалена: близорукость или дальнозоркость? (Дальнозоркость.)
10. Как называется линзовый телескоп: рефлектор или рефрактор? (Рефрактор.)
11. Каково расстояние наилучшего зрения? (25 см.)
12. Какое оптическое тело используется в фотоаппарате для получения четкого изображения? (Линза.)
13. Как называется спектр, который имеет вид четких узких линий на черном фоне? (Линейчатый.)
14. Из скольких цветов состоит сплошной спектр? (7 цветов.)
15. Назовите прибор, с помощью которого исследуется спектральный состав света? (Спектроскоп.)
16. Какие устройства дают спектральное разложение света? (Призма и дифракционная решетка.)
17. Когда мы смотрим телевизор, то видим непрерывное движение предметов, сколько раз происходит смена кадров за одну секунду? (25 раз.)

Ведущая:

– Кто пришел сюда ради веселья?
– Кого вы считаете не достойным играть с вами?

(Из игры выбывает учащиеся с меньшим числом верных ответов)

5-й раунд

Глава 5. Световые кванты

1. Кем созданы основы квантовой теории света? (Планком.)
2. Чему равна энергия кванта света? ()
3. Чему равна постоянная планка? ()
4. Назовите уравнение Эйнштейна для внешнего фотоэффекта. ()
5. Проводимость, обусловленная дополнительными свободными зарядами под действием света? (Фотопроводимость.)
6. Назовите явление наблюдаемое при свечении некоторых твердых тел, например сульфида цинка? (Фосфоресценция.)
7. Назовите вещество способное светится под действием ультрафиолетовых лучей применяемое для создания света близкого по составу к дневному? (Люминофор.)
8. Какой русский, ученый в XIX веке сумел обнаружить и измерить давление света? (Лебедев.)
9. Какими свойствами обладает фотон? (Частица, движущаяся со скоростью света, у которой масса покоя равна нулю.)
10. Назовите формулу давления света? ()
11. В чем заключается эффект Комптона? (В рассеянии длины волны рентгеновского излучения в веществе.)

Ведущая:

– Кто тянет команду ко дну?
– Чей путь победы окончится прямо сейчас?

(Из игры выбывает учащиеся с меньшим числом верных ответов)

6-й раунд

Глава 6. Физика атома

1. Как назвали минимальный электрический заряд? (Элементарный электронный заряд.)
2. Кто открыл электроны? ( Томсон.)
3. Кто впервые измерил заряд электрона? (Милликен.)
4. Кто открыл периодический закон химических элементов? (Менделеев.)
5. Как называют явления испускания атомами невидимых проникающих излучений? (Радиоактивность.)
6. В чем заключается планетарная модель строения атома? Кем она создана? (Положительный заряд находится в центре, вокруг ядра обращаются электроны. Резерфорд.)
7. Для спектра какого элемента Бальмером была получена эмпирическая формула? (Водород.)
8. Кто сформулировал принцип соответствия? (Нильс Бор.)
9. Опыты, каких ученных явились экспериментальным подтверждением правильности основных положений теории Бора? (Франк и Герц.)
10. В каком веке была сделана открытие физических явлений послуживших основой для создания лазера? (XX век.)
11. Чему равна постоянная Ридберга? ()
12. В каком году были сформулированы постулаты Бора? (1913)
13. В каком году Томсон предложил свою модель атома? (1903)
14. Чему равен заряд электрона? ()
15. Чему равна масса электрона? ()

Ведущая: Игра завершена. Всем спасибо!

Учитель: Поздравляем лучшего знатока физики. Спасибо за участие в игре.

Что такое индуктор? — Определение с сайта WhatIs.com

От

Катушка индуктивности — это пассивный электронный компонент, который хранит энергию в виде магнитного поля. В простейшей форме индуктор состоит из проволочной петли или катушки. Индуктивность прямо пропорциональна количеству витков в катушке. Индуктивность также зависит от радиуса катушки и от типа материала, на который намотана катушка.

Для данного радиуса катушки и количества витков у воздушных сердечников наименьшая индуктивность.Такие материалы, как дерево, стекло и пластик, известные как диэлектрические материалы, для обмотки индуктора по сути такие же, как воздух. Ферромагнитные вещества, такие как железо, слоистое железо и порошковое железо, увеличивают индуктивность, которую можно получить с помощью катушки с заданным числом витков. В некоторых случаях это увеличение составляет порядка тысячи раз. Форма сердечника также имеет значение. Тороидальные (кольцевые) сердечники обеспечивают большую индуктивность для данного материала сердечника и количества витков, чем соленоидные (стержневые) сердечники.

Стандартной единицей индуктивности является генри, сокращенно H. Это большая единица. Более распространенными единицами измерения являются микрогенри, сокращенно мкГн (1 мкГн = 10 -6 Гн) и миллигенри, сокращенно мГн (1 мГн = 10 -3 Гн). Иногда используется наногенри (нГн) (1 нГн = 10 -9 Гн).

Сложно изготовить индукторы на микросхемах (ИС). К счастью, резисторы можно заменить индукторами в большинстве микросхем. В некоторых случаях индуктивность можно моделировать с помощью простых электронных схем с использованием транзисторов, резисторов и конденсаторов, изготовленных на микросхемах.

Катушки индуктивности

используются с конденсаторами в различных приложениях беспроводной связи. Катушка индуктивности, подключенная последовательно или параллельно конденсатору, может обеспечить распознавание нежелательных сигналов. Большие индукторы используются в источниках питания электронного оборудования всех типов, включая компьютеры и их периферийные устройства. В этих системах индукторы помогают сглаживать выпрямленный переменный ток в электросети, обеспечивая чистый постоянный ток, подобный батарее.

Последний раз обновлялся в сентябре 2005 г.

Что такое индуктор? | Койлкрафт

Индукторы, катушки и дроссели

Катушка индуктивности — это пассивный электрический компонент, который противодействует резким изменениям тока.Индукторы также известны как катушки или дроссели. Электрический символ индуктора — L.

.

Для чего используется индуктор?

Катушки индуктивности замедляют скачки или скачки тока, временно сохраняя энергию в электромагнитном поле, а затем возвращая ее обратно в цепь.

Индуктор с воздушным или керамическим сердечником Индуктор с ферритовым или железным сердечником

Как индукторы прикреплены к печатным платам?

Катушки индуктивности

для поверхностного монтажа (SM) помещаются на верхнюю часть печатной платы (PCB) на контактные площадки с паяльной пастой, а затем паяются оплавлением.Индукторы со сквозными отверстиями (TH) устанавливаются на верхнюю часть печатной платы с выводами, проходящими через отверстия в плате, а затем припаяны волной на задней стороне.

В каких приложениях используются индукторы?

Катушки индуктивности в основном используются в электрических и электронных устройствах для следующих основных целей:

  1. Подавление, блокировка, ослабление или фильтрация / сглаживание высокочастотного шума в электрических цепях
  2. Хранение и передача энергии в преобразователях мощности (dc-dc или ac-dc)
  3. Создание настроенных осцилляторов или LC (индуктор / конденсатор) «бак» цепей
  4. Согласование импеданса

Что такое дроссель?

Катушка индуктивности, размещенная последовательно (в линию) с проводником, например, проводом или дорожкой печатной платы, блокирует или препятствует изменениям тока и действует как фильтр нижних частот.Поскольку катушки индуктивности ограничивают или блокируют изменения тока, их также называют «дросселями». Например, широкополосный (широкополосный) дроссель смещения в соответствии со смещением постоянного тока усилителя блокирует широкий диапазон высоких частот, позволяя при этом пропускать постоянный ток. Таким образом, дроссель смещения изолирует смещение постоянного тока от радиочастотного сигнала к усилителю.

Федеральная комиссия по связи (FCC) разработала стандарты и сертифицирует электронные устройства, продаваемые или производимые в США, на соответствие требованиям к электромагнитным помехам (EMI).Всемирные организации по стандартизации электромагнитной совместимости (EMC) включают CISPR, IEC, ISO и EN. Нормы FCC являются обязательными и применяются к таким устройствам, как компьютеры, импульсные источники питания, телевизионные приемники, передатчики, а также промышленные, научные и медицинские (ISM) устройства, излучающие радиочастотное излучение. Катушки индуктивности используются в электрических цепях для уменьшения электромагнитных помех за счет ослабления высокочастотного шума, чтобы соответствовать требованиям к электромагнитной совместимости и помехоустойчивости.

Время нарастания тока с индуктором 1 мкГн при 10 В постоянного тока
и 10 Ом нагрузка менее 10 мкс Время нарастания тока с индуктором 10 мкГн при 10 В постоянного тока
и 10 Ом нагрузка больше 40 мкс Рисунок 1

Как я могу улучшить эффективность фильтрации в цепи?

Обычно высокие значения индуктивности необходимы для фильтрации низкочастотного шума, и наоборот: более низкие значения индуктивности используются для фильтрации высокочастотного шума.Высокие значения индуктивности эффективно замедляют время нарастания тока переходных процессов, таких как замыкание переключателя. Графики в Рис. 1 демонстрируют, как индуктор 10 мкГн «сглаживает» время нарастания больше, чем индуктор 1 мкГн.

Катушки индуктивности также можно комбинировать с конденсаторами для создания еще более эффективных LC-фильтров. Существует несколько возможных вариантов настройки LC-фильтра, каждая из которых предполагает компромисс между равномерностью затухания и частотной характеристикой и резкостью спада фильтра.

В этом эталонном проектном документе Coilcraft представлены эталонные конструкции фильтров Баттерворта 3-го порядка и эллиптических ЖК-фильтров 7-го порядка, в которых используются стандартные индукторы для достижения частот среза в диапазоне от 0,3 до 3000 МГц.

Хотя использование высоких значений индуктивности или создание LC-фильтров улучшает фильтрацию, для этого требуется больше места на плате. Поскольку для фильтрации более высоких частот можно использовать более низкие значения индуктивности, переключение на работу на более высокой частоте может позволить использовать катушки индуктивности меньшего размера.

Как индукторы используются в преобразователях мощности?

В импульсных источниках питания индукторы используются для хранения энергии и передачи энергии выходной нагрузке или конденсатору. Индукторы в преобразователях мощности служат для фильтрации «пульсаций» тока на выходе. Высокие значения индуктивности приводят к более низкому току пульсаций, что повышает эффективность и снижает электромагнитные помехи. См. Рисунок 2.

Как индукторы используются в настроенных схемах?

Настроенные схемы используются для передачи или приема сигналов радио- или СВЧ-диапазона.Катушки индуктивности можно комбинировать с конденсаторами для создания настроенных LC-контуров, таких как генераторы.

Преобразователь DC-DC с низким пульсирующим током с индуктором 7,5 мкГн Преобразователь постоянного тока в постоянный с низким уровнем пульсаций с индуктором 75 мкГн Рисунок 2

Как добротность влияет на полосу пропускания LC-контуров?

Q-фактор (Q) — это мера диссипативной характеристики катушки индуктивности. Индукторы с высокой добротностью имеют низкое рассеивание и используются для создания точно настроенных узкополосных схем. Катушки индуктивности с низкой добротностью имеют более высокое рассеивание, что приводит к широкополосным характеристикам.

Что такое собственная резонансная частота индуктора?

Настоящие катушки индуктивности имеют межвитковую емкость обмотки, которая действует как элемент параллельной цепи. Саморезонансная частота (SRF) катушки индуктивности — это частота, при которой индуктивное реактивное сопротивление равно по величине емкостному реактивному сопротивлению обмоток. В SRF индуктивный и емкостной фазовые углы компенсируются, и импеданс фактически является чисто резистивным. Величина импеданса увеличивается с частотой до собственной резонансной частоты (SRF), где импеданс катушки индуктивности достигает максимального значения.На частотах выше SRF сопротивление уменьшается с увеличением частоты.

Импеданс (Z) — это характеристика электрических компонентов, которая включает комбинацию вектора сопротивления и фазы. Сопротивление имеет свойство рассеивания: энергия используется, а не восстанавливается. Фаза — это задержка между приложенным напряжением на компоненте и током, протекающим через него, чаще всего выражается как угол в градусах (°) или радианах. И сопротивление переменному току, и фаза катушек индуктивности меняются в зависимости от частоты.

Как используются индукторы для согласования импеданса?

Согласование импеданса обычно включает в себя согласование импеданса источника питания с импедансом электрической нагрузки. Максимальная мощность передается от источника к нагрузке, когда полное сопротивление нагрузки согласуется с сопротивлением источника, что повышает эффективность схемы. Если нагрузка является емкостной по сравнению с источником, можно использовать катушки индуктивности для противодействия емкости нагрузки и, таким образом, согласования полного сопротивления.

Какие типы индукторов производит компания Coilcraft?

Coilcraft разрабатывает и производит стандартные индукторы различных размеров и конструкций для удовлетворения разнообразных требований к фильтрации, настройке и согласованию импеданса.

Примечания к приложению

Основные факты об индукторах [Урок 1] Обзор индукторов — «Как работают индукторы?»

Направляющая индуктора

Катушка индуктивности — это пассивный электронный компонент, способный накапливать электрическую энергию в виде магнитной энергии.По сути, он использует проводник, намотанный на катушку, и когда электричество течет в катушку слева направо, это создает магнитное поле в направлении по часовой стрелке.

Ниже представлено уравнение индуктивности катушки индуктивности. Чем больше витков намотано на сердечник, тем сильнее создается магнитное поле. Сильное магнитное поле также создается за счет увеличения площади поперечного сечения индуктора или изменения сердечника индуктора.

Давайте теперь предположим, что через катушку индуктивности протекает переменный ток. «AC» (переменный ток) относится к току, уровень и направление которого циклически меняются с течением времени. Когда ток приближается к индуктору, магнитное поле, создаваемое этим током, пересекает другие обмотки, вызывая индуцированное напряжение и, таким образом, предотвращая любые изменения уровня тока. Если ток вот-вот возрастет внезапно, электродвижущая сила генерируется в направлении, противоположном току, то есть в том направлении, в котором ток уменьшается, что предотвращает любое увеличение тока.И наоборот, если ток вот-вот упадет, электродвижущая сила генерируется в том направлении, в котором ток увеличивается.

Эти эффекты индуцированного напряжения возникают даже тогда, когда направление тока меняется на противоположное. Перед преодолением индуцированного напряжения, которое пытается заблокировать ток, направление тока меняется на противоположное, чтобы ток не протекал.

Уровень тока остается неизменным, когда постоянный ток течет к катушке индуктивности, поэтому индуцированное напряжение не создается, и можно считать, что возникает закороченное состояние.Другими словами, индуктор — это компонент, который позволяет постоянному току, но не переменному току, проходить через него.

  • Катушка индуктивности накапливает электрическую энергию в виде магнитной энергии.
  • Катушка индуктивности не пропускает через себя переменный ток, но пропускает через нее постоянный ток.

Свойства индукторов используются во множестве различных приложений. Существует множество различных типов индукторов, и в следующем уроке будут описаны приложения, для которых индукторы лучше всего подходят.

Ответственное лицо: Murata Manufacturing Co., Ltd. T.K

Сопутствующие товары

Катушки индуктивности

Статьи по теме

Будьте в курсе!

Получайте электронные письма от Мураты с последними обновлениями на этом сайте.
Информационный бюллетень Murata (электронный информационный бюллетень)

mail_outline

Оцените свою трехмерную конструкцию индуктора с помощью COMSOL Multiphysics

Индуктивные устройства демонстрируют ряд электромагнитных эффектов, которые необходимо учитывать при их использовании в любом приложении.С помощью инструментов, предоставляемых модулем AC / DC в COMSOL Multiphysics, вы можете легко смоделировать и спроектировать катушку индуктивности, а также рассчитать характеристики устройства, необходимые для вашего приложения.

Основные физические принципы индуктора

Катушка индуктивности, в простейшем случае, представляет собой проводящий провод (обмотка или катушка ), намотанный на кусок обычно магнитного материала (сердечник ). Его действие зависит от концепции индуктивности , посредством которой магнитное поле, создаваемое вокруг индуктора, противодействует изменению тока, протекающего через устройство.


Простой индуктор, состоящий из медной обмотки вокруг железного сердечника. Красные стрелки показывают направление тока, а синие стрелки показывают магнитное поле вне сердечника. Изменение тока также изменяет магнитный поток через обмотку, создавая потенциал на обмотке, который противодействует изменению тока.

Индуктивность — это параметр, который измеряет степень сопротивления изменению протекания тока и обычно обозначается буквой L, что полностью характеризует идеальную катушку индуктивности.К сожалению, мы не живем в идеальном мире, и настоящие катушки индуктивности также демонстрируют резистивные эффекты (важные на низких частотах и ​​характеризующиеся сопротивлением R) и емкостные эффекты (важные на высоких частотах и ​​характеризующиеся емкостью C), которые вызывают саморезонанс. Фактически, трехмерную катушку индуктивности часто можно хорошо понять с помощью модели цепи RLC или ее некоторого расширения.


Эквивалентная схема, которая может имитировать поведение реальной катушки индуктивности.

Самостоятельно или в сочетании с другими элементами схемы, такими как конденсаторы или резисторы, свойство индуктивности может применяться во всех видах приложений.При использовании переменного тока только катушка индуктивности может функционировать как фильтр нижних частот или последовательно с конденсатором как резонансный фильтр или полосовой фильтр . Индукторы также играют важную роль во многих ключевых областях повседневной жизни, например, в импульсных источниках питания и согласующих схемах, подключенных к радиочастотным антеннам. Светофор, который удобно меняется по мере приближения, вполне может быть индуктивным датчиком, помогающим вам в пути!

Оптимизация конструкции индуктора по характеристикам

Если ваше устройство включает индуктор, важно знать некоторые характеристики индуктора, чтобы полностью понять, как ваше устройство работает в целом.Ключевые параметры включают индуктивность и могут также включать сопротивление; емкость; резонансная частота; и Q-фактор , ширина пика вокруг резонансной частоты. Эти параметры определяют такие факторы, как частота среза или полоса пропускания для приложений фильтрации или просто реактивное сопротивление согласующей схемы.

Другая потенциальная проблема при использовании индукторов — это электромагнитные помехи (EMI) или электромагнитная совместимость (EMC).Поскольку индукторы создают магнитное поле вокруг своих катушек, вам может потребоваться знать, какое влияние это окажет на другие компоненты или устройства, находящиеся поблизости, тем более, что современные схемы стали более плотно упакованными.

Существуют грубые аналитические или эмпирические формулы для описания этих параметров RLC, но такие формулы не могут обеспечить высокую точность, необходимую для проектирования современных устройств. Это особенно верно, когда форма устройства отклоняется от тех немногих форм, которые можно легко обрабатывать аналитически, таких как кубоиды, цилиндры, спирали и торы.Форма и спад магнитного поля вокруг индуктивного устройства для целей EMI / EMC описаны хуже.

Кроме того, вы можете сделать сердечник индуктора из нелинейного магнитного материала, чтобы увеличить индуктивность и ограничить окружающее магнитное поле. Это добавило бы еще один уровень сложности расчетам, а значит, еще один уровень аналитического или эмпирического приближения, что поставило бы под сомнение результат. Таким образом, чтобы получить действительно точную характеристику трехмерного индуктивного устройства, вычислительное моделирование предлагает более надежное решение.

Моделирование трехмерного индуктора в COMSOL Multiphysics

COMSOL Multiphysics предоставляет все инструменты, необходимые для полного определения характеристик вашего индуктора в том приложении, для которого он используется. Учебная модель «Моделирование 3D-индуктора» из галереи приложений, которая также является вводной моделью для модуля переменного / постоянного тока, дает хорошую иллюстрацию некоторых ключевых функций программного обеспечения. Это снимает напряжение с изучения того, как определять характеристики и проектировать катушки индуктивности.


Геометрия индуктора для нашей конечно-элементной модели.

В реальном устройстве ток может проходить через катушку индуктивности различными способами. Можно подать заданное напряжение, ток или мощность. Это может быть постоянное значение, оно может колебаться или более сложным образом зависеть от времени. В этом примере применения однооборотная катушка и функция сосредоточенного порта (используемые на низких и высоких частотах соответственно) управляют током в проводниках, показывая, как можно реалистично управлять своими катушками, неважно приложение.

Этапы моделирования 3D-индуктора в COMSOL Multiphysics.

COMSOL Multiphysics позволяет не только легко настраивать модели цепей, что можно сделать в самом программном обеспечении или импортируя список цепей SPICE, но также и с последующим подключением этих моделей цепей к моделям конечных элементов. Вы можете подключить катушку индуктивности к управляющей цепи, используя встроенные функции соединения между полной трехмерной моделью катушки индуктивности и моделью цепи с минимальными усилиями, поскольку COMSOL Multiphysics определяет, какие части могут быть соединены вместе, и представляет вам эти возможности в удобный ящик.



Когда вы выбираете входную функцию из выпадающего списка, вывод индуктора (выделен синим) автоматически подключается к модели схемы.

На высоких частотах (на которых может работать катушка индуктивности) хорошо известно, что ток в проводнике удерживается близко к его поверхности из-за скин-эффекта. В примере приложения этот эффект включен с помощью граничного условия «Импеданс» в COMSOL Multiphysics. Разрешение течения тока в тонком слое — это дорогостоящая процедура, поэтому упрощение ее в качестве граничного условия экономит ваше время и позволяет быстрее исследовать вашу конструкцию.


Токи (Am -2 ), генерируемые на поверхности катушки с высокой частотой. Обратите внимание, что неоднородный характер течения полностью улавливается.

Добротность самого индуктора в решающей степени зависит от свойств материала его сердечника, особенно от потерь. Гибкость COMSOL Multiphysics позволяет изменять свойства материала по мере необходимости, чтобы учесть эти потери в вашей модели. Любые потери на вихревые токи автоматически включаются в расчет, и здесь диэлектрические потери в сердечнике добавляются к модели через определяемый пользователем мнимый вклад в диэлектрическую проницаемость, ε r .Используя тот же простой подход, вы можете добавить магнитные потери к своей модели через комплексную магнитную проницаемость, μ r .

Постобработка и анализ результатов модели индуктора

Если у вас есть руководство, которое нужно привлечь к себе, или клиентов, которых нужно впечатлить, вы можете создать мощные графики результатов всего за несколько щелчков мышью, используя встроенные возможности постобработки COMSOL Multiphysics. Программное обеспечение автоматически рассчитывает и предоставляет доступ к переменным, которые вы оцениваете, чтобы оценить конструкцию индуктора, например, магнитное поле, токи и скорость потерь.Пример применения 3D-индуктора демонстрирует, как настроить подробное изображение, показанное ниже.


В сердечнике плотность магнитного потока на поверхности (T) выше там, где материал тоньше, поскольку он несет примерно постоянный поток. В катушке на проводнике (V) создается местный потенциал.

Потратив немного больше времени, вы можете расширить свои результаты, включив в них некоторые другие доступные типы графиков, такие как графики Streamline или Arrow .


В сердечнике плотность магнитного потока отображается на поверхности (T). В катушке линии тока плотности тока (Am -2 ) показывают более высокую плотность тока внутри изогнутых частей. Стрелки в окружающих областях показывают направление магнитного поля.

Наконец, для конечной цели вашей модели вам потребуются значения импеданса и резонансные частоты для вашей катушки индуктивности. Одной из многих переменных, которые COMSOL Multiphysics автоматически вычисляет, является точное полное сопротивление катушки индуктивности на каждой частоте, поэтому эти параметры можно легко построить на графике.Используя встроенные операторы real и imag , вы можете построить график реальной (резистивной) и мнимой (индуктивной / емкостной) частей импеданса, где можно легко увидеть резонанс.



Реальная (левая) и мнимая (правая) части импеданса Z, показывающие резонанс и переключение между индуктивным и емкостным режимами.

В мнимой части вы также можете видеть, что знак меняется с положительного на отрицательный при прохождении резонансной частоты, что означает, что устройство переходит от преимущественно индуктивного действия к преимущественно емкостному на высоких частотах, что в точности соответствует ожиданиям.

Расширение модели индуктора с учетом тепловых эффектов

COMSOL Multiphysics разработан для простой интеграции различных физических эффектов в модель — это «мультифизический» аспект программного обеспечения. Одним из способов расширения этой модели индуктора является рассмотрение его электромагнитного нагрева. Токи проводимости в катушке, вихревые токи и диэлектрические / магнитные потери в сердечнике — все это создает тепло, которое распространяется через металлические компоненты с высокой теплопроводностью и, возможно, в окружающее устройство и печатную плату.Используя интерфейс Induction Heating , вы можете легко добавить расчет скорости нагрева и распределения температуры в вашем индуктивном устройстве.

Подробнее о моделировании индукторов в COMSOL Multiphysics

  • Ознакомьтесь с другой учебной моделью индуктора в галерее приложений:
  • Узнайте о том, как другие применяли COMSOL Multiphysics для разработки своих индуктивных устройств:

AC Inductive Circuits

Индуктор обычно представляет собой катушку с проволокой, которая создает переменное магнитное поле вокруг нее, когда через нее протекает переменный ток.Индуктивность — это свойство катушки индуктивности, которая препятствует изменению тока. Измеряется в Генри. Из-за этой индуктивности в катушке индуцируется обратная ЭДС при воздействии на нее переменного тока.

Согласно закону Ленца, эта ЭДС противодействует изменению силы тока. Следовательно, приложенное напряжение должно преодолевать только эту обратную ЭДС, потому что в цепи нет сопротивления. Таким образом, приложенное напряжение и обратная ЭДС должны быть равными и противоположными, чтобы ток протекал по цепи.

Поведение цепи переменного тока с индуктором полностью отличается от цепи постоянного тока. При этом ток, протекающий через катушку, зависит не только от индуктивности, но и от частоты переменного источника. Обсудим вкратце поведение цепи переменного тока при индуктивных нагрузках.

Переменный ток, приложенный через чистый индуктор

Чистый индуктор не имеет сопротивления в обмотке катушки, но имеет только индуктивность. Это свойство индуктивности проявляется во всех двигателях, трансформаторах и генераторах (с некоторым сопротивлением в катушке).На рисунке ниже показана чисто индуктивная цепь с источником переменного напряжения и соответствующие формы сигналов.

Пусть приложенное напряжение, v = V m sin ωt. Как указано выше, индуцированная ЭДС равна приложенному напряжению и противоположна ему, т.е. v = — e

Где e — обратная ЭДС и равна –L di / dt

Подставляя выражение для ЭДС, получаем

v = L di / dt

V m sin ωt = L di / dt

di = (V m / L) sin ωt dt

Применяя интегрирование с обеих сторон, мы получаем

i = ( V m / L) ∫ sin ωt dt

= (V m / ωL) (- cos ωt)

i = (V m / wL) (sin ωt — π / 2)

Когда (sin ωt — π / 2) равно единице, ток, протекающий по цепи, будет максимальным.Таким образом,

im = (V m / ωL)

Тогда текущее уравнение принимает вид

i = i m sin (ωt — π / 2)

Где i m = (V m / ωL)

Из приведенных выше выражений тока и напряжения ясно, что ток отстает от напряжения на 900. Следовательно, в чистой индуктивной цепи ток находится в квадратуре с напряжением, как показано в формах сигналов на приведенном выше рисунке.

Это означает, что когда изменение тока является максимальным (при токе, проходящем через ноль), напряжение, индуцированное на катушке индуктивности, является максимальным.Точно так же при максимальных значениях тока, когда ток не изменяется, индуцированное напряжение на катушке индуктивности будет равно нулю.

Таким образом, напряжение на катушке индуктивности опережает ток через эту катушку индуктивности на (четверть) цикла. Векторная диаграмма чисто индуктивной цепи переменного тока приведена ниже.

Индуктивное реактивное сопротивление

Из приведенного выше вывода уравнение максимального тока дается как

i м = (V m / ωL)

ωL = V m / i m

Это отношение напряжения к току является противодействием индуктивной цепи протеканию тока.Эта величина wL называется индуктивным реактивным сопротивлением и обозначается XL и измеряется в Ом.

Индуктивное реактивное сопротивление цепи переменного тока можно представить как

XL = ωL = 2 (fL (так как ω = 2Πf)

Где XL — индуктивное реактивное сопротивление в Ом

f — частота напряжения питания

L — индуктивность катушки в Генри

Приведенное выше уравнение говорит о том, что при увеличении частоты входного питания изменяется и скорость изменения тока.Следовательно, наведенная ЭДС (или напряжение реакции) на катушке индуктивности будет увеличиваться.

В результате чистый ток, протекающий через катушку индуктивности, будет уменьшен. Следует сделать вывод, что реактивное сопротивление катушки индуктивности изменяется линейно с частотой источника питания, как показано на рисунке.

Мощность и коэффициент мощности в индуктивной цепи переменного тока

Мощность в цепи переменного тока является произведением мгновенного напряжения и тока. Это можно представить как

P = v × i

P = V m sin ωt × I m sin (ωt — 90)

Если интегрировать по циклу, мы получим

P = V m sin ωt × I м sin (ωt — 90)

P = 1 / 2π (∫ 0 V м sin ωt × I м sin (ωt — 90) dωt)

= (V m I m / 2π) (∫ 0 sin ωt × (- cos ωt) dwt)

= (V m I m / 2π) (∫ 0 (- sin 2 ωt) / 2 dwt)

= (V m I m / 8π) (cos 4π — cos 0)

= (V m I m / 8π ) (1 — 1)

P = 0

Средняя мощность в чистой катушке индуктивности всегда равна нулю, потому что количество энергии, полученное от источника за полупериод, возвращается к источнику в следующем полупериоде.

На рисунке ниже показана кривая мощности индуктивной цепи переменного тока, в которой положительная мощность равна отрицательной мощности, поэтому результирующая мощность за цикл равна нулю. Это ясно объясняет, что чистая индуктивность не потребляет энергии.

В этой схеме ток также синусоидальный, но отстает от напряжения на 900. Поскольку ток отстает от напряжения на 90 0 , разность фаз θ равна 90 0 . Тогда

Коэффициент мощности cos 90 = 0

Коэффициент мощности в чисто индуктивной цепи равен нулю, т.е.е., чисто отстающий коэффициент мощности.

Серия RL Цепь

Как мы знаем, физической цепи чистой индуктивности не существует, потому что каждая катушка наряду с индуктивностью имеет некоторое сопротивление обмотки. В такой схеме сопротивление считается последовательным элементом индуктора.

Рассмотрим рисунок ниже, на котором чистое сопротивление последовательно соединено с чистой индуктивностью. Эта последовательная комбинация подключена к источнику переменного тока с напряжением v = V m sin ωt.

В цепи последовательно R L напряжение на катушке индуктивности не совпадает по фазе с током, протекающим по цепи, и напряжением на сопротивлении, как показано на рисунке выше.Индуцированное напряжение в катушке индуктивности препятствует прохождению тока, и, следовательно, V L опережает ток I и падает на сопротивлении V R на 90 0 .

Пусть I — ток, протекающий по цепи, V L и V R — это падения напряжения на индуктивности и сопротивлении соответственно.

Напряжение на резисторе, В R = I R

Напряжение на катушке индуктивности, В L = I × XL (где XL = 2πfL)

Из приведенного выше вектора

В = √ (V R 2 + V L 2 ) = √ (IR) 2 + (I XL) 2 )

= I √ (R 2 + XL 2 ) = I Z

Где Z — полное сопротивление в последовательной цепи R L , равное √ (R 2 + XL 2 ).

Треугольник полного сопротивления

Сопротивление, создаваемое цепью переменного тока потоку синусоидального тока, называется импедансом. Его также можно определить как отношение синусоидального напряжения к току. Обозначается буквой Z и измеряется в Ом.

Из векторной диаграммы серии RL,

tan ϕ = V L / V R = XL / R

cos ϕ = V R / V = ​​R / Z

sin ϕ = V L / V = ​​XL / Z

Если все стороны треугольника, полученного в последовательной цепи X L , разделить на ток, мы получим треугольник импеданса, как показано на рисунке.Из этого треугольника компоненты R, X L и Z могут быть выражены как

R = Z cos ϕ

XL = Z sin ϕ

Z = √ (R 2 + XL 2 )

And ϕ = tan-1 (XL / R)

Пример

Найдите выражение для тока, а также вычислите мощность последовательной цепи RL с R = 50 Ом и L = 0,159 Гн, возбуждаемой напряжением v = 283 sin 100πt .

Индуктивное реактивное сопротивление, XL = 2πfL = 100π × 0,159

= 49,95 Ом

Z = R + j XL = 50 + j49.95

Преобразуя в полярную форму, получаем Z = 70,675 ∠44,97 Ом

Ток, i = v / Z = (283 sin (100πt — 44,97)) / 70,675

i = 4 sin (100πt — π / 4) A

P = VI cos θ

= (283 / √2) (4 / √2) cos 44,97

= 400,43 A

Магнитный поток, индукция и закон Фарадея

Индуцированные ЭДС и магнитный поток

Закон индукции Фарадея гласит, что электродвижущая сила индуцируется изменением магнитного потока.

Цели обучения

Объясните взаимосвязь между магнитным полем и электродвижущей силой

Основные выводы

Ключевые моменты
  • Это изменение потока магнитного поля, которое приводит к возникновению электродвижущей силы (или напряжения).
  • Магнитный поток (часто обозначаемый Φ или Φ B ), проходящий через поверхность, является составляющей магнитного поля, проходящего через эту поверхность.
  • В самом общем виде магнитный поток определяется как [латекс] \ Phi _ {\ text {B}} = \ iint _ {\ text {A}} \ mathbf {\ text {B}} \ cdot \ text {d} \ mathbf {\ text {A}} [/ latex].Это интеграл (сумма) всего магнитного поля, проходящего через бесконечно малые элементы площади dA.
Ключевые термины
  • площадь вектора : вектор, величина которого соответствует рассматриваемой области, а направление перпендикулярно площади поверхности.
  • гальванометр : аналоговое измерительное устройство, обозначенное буквой G, которое измеряет ток, используя отклонение стрелки, вызванное силой магнитного поля, действующей на провод с током.

Индуцированная ЭДС

Аппарат, использованный Фарадеем для демонстрации того, что магнитные поля могут создавать токи, показан на следующем рисунке. Когда переключатель замкнут, магнитное поле создается в катушке в верхней части железного кольца и передается (или направляется) на катушку в нижней части кольца. Гальванометр используется для обнаружения любого тока, наведенного в отдельной катушке внизу.

Аппарат Фарадея : Это аппарат Фарадея для демонстрации того, что магнитное поле может производить ток.Изменение поля, создаваемого верхней катушкой, вызывает ЭДС и, следовательно, ток в нижней катушке. Когда переключатель разомкнут и замкнут, гальванометр регистрирует токи в противоположных направлениях. Когда переключатель остается замкнутым или разомкнутым, через гальванометр не течет ток.

Было обнаружено, что каждый раз, когда переключатель замыкается, гальванометр обнаруживает ток в одном направлении в катушке внизу. Каждый раз при размыкании переключателя гальванометр обнаруживает ток в противоположном направлении.Интересно, что если переключатель остается замкнутым или разомкнутым в течение какого-то времени, через гальванометр нет тока. Замыкание и размыкание переключателя индуцирует ток. Это изменение магнитного поля, которое создает ток. Более важным, чем текущий ток, является вызывающая его электродвижущая сила (ЭДС). Ток является результатом ЭДС, индуцированной изменяющимся магнитным полем, независимо от того, есть ли путь для протекания тока.

Магнитный поток

Магнитный поток (часто обозначаемый Φ или Φ B ), проходящий через поверхность, является составляющей магнитного поля, проходящего через эту поверхность.Магнитный поток через некоторую поверхность пропорционален количеству силовых линий, проходящих через эту поверхность. Магнитный поток, проходящий через поверхность с векторной площадью А, равен

.

[латекс] \ Phi_ \ text {B} = \ mathbf {\ text {B}} \ cdot \ mathbf {\ text {A}} = \ text {BA} \ cos \ theta [/ latex],

, где B — величина магнитного поля (в Тесла, Тл), A — площадь поверхности, а θ — угол между силовыми линиями магнитного поля и нормалью (перпендикулярно) к A.

Для переменного магнитного поля мы сначала рассмотрим магнитный поток [латекс] \ text {d} \ Phi _ \ text {B} [/ latex] через бесконечно малый элемент площади dA, где мы можем считать поле постоянным:

Изменяющееся магнитное поле : Каждая точка на поверхности связана с направлением, называемым нормалью к поверхности; магнитный поток, проходящий через точку, тогда является составляющей магнитного поля вдоль этого нормального направления.

[латекс] \ text {d} \ Phi_ \ text {B} = \ mathbf {\ text {B}} \ cdot \ text {d} \ mathbf {\ text {A}} [/ latex]

Общая поверхность A затем может быть разбита на бесконечно малые элементы, и тогда полный магнитный поток через поверхность равен интегралу поверхности

[латекс] \ Phi_ \ text {B} = \ iint_ \ text {A} \ mathbf {\ text {B}} \ cdot \ text {d} \ mathbf {\ text {A}} [/ latex].

Закон индукции Фарадея и закон Ленца

Закон индукции Фарадея гласит, что ЭДС, индуцированная изменением магнитного потока, равна [латексу] \ text {EMF} = — \ text {N} \ frac {\ Delta \ Phi} {\ Delta \ text {t}} [ / латекс], когда поток изменяется на Δ за время Δt.

Цели обучения

Выразите закон индукции Фарадея в форме уравнения

Основные выводы

Ключевые моменты
  • Минус в законе Фарадея означает, что ЭДС создает ток I и магнитное поле B, которые противодействуют изменению потока Δ, известному как закон Ленца.
  • Закон индукции Фарадея является основным принципом работы трансформаторов, индукторов и многих типов электродвигателей, генераторов и соленоидов.
  • Закон Фарадея гласит, что ЭДС, вызванная изменением магнитного потока, зависит от изменения магнитного потока Δ, времени Δt и количества витков катушек.
Ключевые термины
  • электродвижущая сила : (ЭДС) — напряжение, генерируемое батареей или магнитной силой в соответствии с законом Фарадея.Она измеряется в вольтах, а не в ньютонах, и поэтому на самом деле не является силой.
  • соленоид : Катушка с проволокой, которая действует как магнит, когда через нее протекает электрический ток.
  • поток : Скорость передачи энергии (или другой физической величины) через данную поверхность, в частности электрического или магнитного потока.

Закон индукции Фарадея

Закон индукции Фарадея — это основной закон электромагнетизма, который предсказывает, как магнитное поле будет взаимодействовать с электрической цепью, создавая электродвижущую силу (ЭДС).Это основной принцип работы трансформаторов, индукторов и многих типов электродвигателей, генераторов и соленоидов.

Эксперименты Фарадея показали, что ЭДС, вызванная изменением магнитного потока, зависит только от нескольких факторов. Во-первых, ЭДС прямо пропорциональна изменению потока Δ. Во-вторых, ЭДС является наибольшей, когда изменение во времени Δt наименьшее, то есть ЭДС обратно пропорциональна Δt. Наконец, если катушка имеет N витков, будет создаваться ЭДС, которая в N раз больше, чем для одиночной катушки, так что ЭДС прямо пропорциональна N.Уравнение для ЭДС, вызванной изменением магнитного потока, равно

[латекс] \ text {EMF} = — \ text {N} \ frac {\ Delta \ Phi} {\ Delta \ text {t}} [/ latex].

Это соотношение известно как закон индукции Фарадея. Единицы измерения ЭДС, как обычно, — вольты.

Закон Ленца

Знак минус в законе индукции Фарадея очень важен. Минус означает, что ЭДС создает ток I и магнитное поле B, которые противодействуют изменению потока Δ, известному как закон Ленца. Направление (обозначенное знаком минус) ЭМП настолько важно, что оно названо законом Ленца в честь русского Генриха Ленца (1804–1865), который, подобно Фарадею и Генри, независимо исследовал аспекты индукции.Фарадей знал о направлении, но Ленц указал его, поэтому ему приписывают это открытие.

Закон Ленца : (a) Когда стержневой магнит вставляется в катушку, сила магнитного поля в катушке увеличивается. Ток, наведенный в катушке, создает другое поле в направлении, противоположном стержневому магниту, чтобы противодействовать увеличению. Это один из аспектов закона Ленца: индукция препятствует любому изменению потока. (b) и (c) — две другие ситуации. Убедитесь сами, что показанное направление индуцированной катушки B действительно противостоит изменению магнитного потока и что показанное направление тока согласуется с правилом правой руки.

Энергосбережение

Закон Ленца является проявлением сохранения энергии. Индуцированная ЭДС создает ток, который противодействует изменению потока, потому что изменение потока означает изменение энергии. Энергия может входить или уходить, но не мгновенно. Закон Ленца — это следствие. Когда изменение начинается, закон говорит, что индукция противодействует и, таким образом, замедляет изменение. Фактически, если бы индуцированная ЭДС была в том же направлении, что и изменение потока, была бы положительная обратная связь, которая не давала бы нам бесплатную энергию из любого видимого источника — закон сохранения энергии был бы нарушен.

Движение ЭДС

Движение в магнитном поле, которое является стационарным относительно Земли, вызывает ЭДС движения (электродвижущую силу).

Цели обучения

Определить процесс, вызывающий двигательную электродвижущую силу

Основные выводы

Ключевые моменты
  • Закон индукции Фарадея можно использовать для расчета ЭДС движения, когда изменение магнитного потока вызвано движущимся элементом в системе.
  • То, что движущееся магнитное поле создает электрическое поле (и, наоборот, движущееся электрическое поле создает магнитное поле), является частью причины, по которой электрические и магнитные силы теперь рассматриваются как разные проявления одной и той же силы.
  • Любое изменение магнитного потока индуцирует электродвижущую силу (ЭДС), противодействующую этому изменению — процесс, известный как индукция. Движение — одна из основных причин индукции.
Ключевые термины
  • электродвижущая сила : (ЭДС) — напряжение, генерируемое батареей или магнитной силой в соответствии с законом Фарадея. Она измеряется в вольтах, а не в ньютонах, и поэтому на самом деле не является силой.
  • магнитный поток : Мера силы магнитного поля в заданной области.
  • индукция : Генерация электрического тока изменяющимся магнитным полем.

Как было замечено в предыдущих атомах, любое изменение магнитного потока индуцирует электродвижущую силу (ЭДС), противодействующую этому изменению — процесс, известный как индукция. Движение — одна из основных причин индукции. Например, магнит, перемещенный к катушке, индуцирует ЭДС, а катушка, перемещенная к магниту, создает аналогичную ЭДС. В этом атоме мы концентрируемся на движении в магнитном поле, которое является стационарным относительно Земли, производя то, что в общих чертах называется ЭДС движения.

ЭДС движения

Рассмотрим ситуацию, показанную на. Стержень перемещается со скоростью v по паре проводящих рельсов, разделенных расстоянием в однородном магнитном поле B. Рельсы неподвижны относительно B и соединены с неподвижным резистором R ( резистором может быть что угодно от лампочки до вольтметра). Учтите площадь, ограниченную подвижным стержнем, направляющими и резистором. B перпендикулярно этой области, и площадь увеличивается по мере перемещения стержня. Таким образом, магнитный поток между рельсами, стержнем и резистором увеличивается.Когда поток изменяется, ЭДС индуцируется согласно закону индукции Фарадея.

ЭДС движения : (a) ЭДС движения = Bℓv индуцируется между рельсами, когда этот стержень перемещается вправо в однородном магнитном поле. Магнитное поле B направлено внутрь страницы, перпендикулярно движущемуся стержню и рельсам и, следовательно, к области, окружающей их. (б) Закон Ленца дает направление индуцированного поля и тока, а также полярность наведенной ЭДС. Поскольку поток увеличивается, индуцированное поле направлено в противоположном направлении или за пределы страницы.Правило правой руки дает указанное направление тока, и полярность стержня будет управлять таким током.

Чтобы найти величину ЭДС, индуцированной вдоль движущегося стержня, мы используем закон индукции Фарадея без знака:

[латекс] \ text {EMF} = \ text {N} \ frac {\ Delta \ Phi} {\ Delta \ text {t}} [/ latex].

В этом уравнении N = 1 и поток Φ = BAcosθ. Имеем θ = 0º и cosθ = 1, так как B перпендикулярно A. Теперь Δ = Δ (BA) = BΔA, поскольку B однородна. Отметим, что площадь, заметаемая стержнем, равна ΔA = ℓx.Ввод этих величин в выражение для ЭДС дает:

[латекс] \ text {EMF} = \ frac {\ text {B} \ Delta \ text {A}} {\ Delta \ text {t}} = \ text {B} \ frac {\ text {l} \ Дельта \ text {x}} {\ Delta \ text {t}} = \ text {Blv} [/ latex].

Чтобы найти направление индуцированного поля, направление тока и полярность наведенной ЭДС, мы применяем закон Ленца, как объяснено в Законе индукции Фарадея: Закон Ленца. Как видно на рис. 1 (b), уровень освещенности увеличивается, так как увеличивается закрытая площадь.Таким образом, индуцированное поле должно противостоять существующему и быть вне страницы. (Правило правой руки требует, чтобы я вращался против часовой стрелки, что, в свою очередь, означает, что вершина стержня положительна, как показано.)

Электрическое поле против магнитного поля

Между электрической и магнитной силой существует множество связей. То, что движущееся магнитное поле создает электрическое поле (и, наоборот, движущееся электрическое поле создает магнитное поле), является частью причины, по которой электрические и магнитные силы теперь рассматриваются как различных проявлений одной и той же силы (впервые замечено Альбертом Эйнштейном) .Это классическое объединение электрических и магнитных сил в так называемую электромагнитную силу является источником вдохновения для современных усилий по объединению других основных сил.

Обратная ЭДС, вихревые токи и магнитное демпфирование

Обратная ЭДС, вихревые токи и магнитное затухание — все это происходит из-за наведенной ЭДС и может быть объяснено законом индукции Фарадея.

Цели обучения

Объясните взаимосвязь между двигательной электродвижущей силой, вихревыми токами и магнитным демпфированием

Основные выводы

Ключевые моменты
  • Входной ЭДС, которая питает двигатель, может противодействовать собственная ЭДС двигателя, называемая обратной ЭДС двигателя.
  • Если ЭДС движения может вызвать токовую петлю в проводнике, ток называется вихревым током.
  • Вихревые токи могут вызывать значительное сопротивление движению, называемое магнитным демпфированием.
Ключевые термины
  • электродвижущая сила : (ЭДС) — напряжение, генерируемое батареей или магнитной силой в соответствии с законом Фарадея. Она измеряется в вольтах, а не в ньютонах, и поэтому на самом деле не является силой.
  • Закон индукции Фарадея : Основной закон электромагнетизма, который предсказывает, как магнитное поле будет взаимодействовать с электрической цепью, создавая электродвижущую силу (ЭДС).

Задняя ЭДС

Двигатели и генераторы очень похожи. (Прочтите наши атомы в разделах «Электрические генераторы» и «Электродвигатели».) Генераторы преобразуют механическую энергию в электрическую, а двигатели преобразуют электрическую энергию в механическую. Кроме того, двигатели и генераторы имеют одинаковую конструкцию. Когда катушка двигателя поворачивается, магнитный поток изменяется, и возникает электродвижущая сила (ЭДС), соответствующая закону индукции Фарадея. Таким образом, двигатель действует как генератор всякий раз, когда его катушка вращается.Это произойдет независимо от того, поворачивается ли вал под действием внешнего источника, например ременной передачи, или под действием самого двигателя. То есть, когда двигатель выполняет работу и его вал вращается, возникает ЭДС. Закон Ленца говорит нам, что наведенная ЭДС противодействует любому изменению, так что входной ЭДС, питающей двигатель, будет противодействовать самогенерируемая ЭДС двигателя, называемая обратной ЭДС двигателя.

Вихретоковый

Как обсуждалось в разделе «ЭДС движения», ЭДС движения индуцируется, когда проводник движется в магнитном поле или когда магнитное поле движется относительно проводника.Если подвижная ЭДС может вызвать токовую петлю в проводнике, мы называем этот ток вихревым. Вихревые токи могут вызывать значительное сопротивление движению, называемое магнитным затуханием.

Рассмотрим устройство, показанное на рисунке, которое раскачивает маятник между полюсами сильного магнита. Если боб металлический, то при входе в поле и выходе из поля он испытывает значительное сопротивление, что быстро гасит движение. Однако, если боб представляет собой металлическую пластину с прорезями, как показано на (b), эффект от магнита будет гораздо меньше.Заметного воздействия на боб из изолятора не наблюдается.

Устройство для исследования вихревых токов и магнитного затухания : Обычное демонстрационное устройство для изучения вихревых токов и магнитного затухания. (а) Движение металлического маятника, раскачивающегося между полюсами магнита, быстро затухает под действием вихревых токов. (b) Имеется незначительное влияние на движение металлического боба с прорезями, что означает, что вихревые токи становятся менее эффективными. (c) На непроводящем бобе также отсутствует магнитное затухание, поскольку вихревые токи чрезвычайно малы.

показывает, что происходит с металлической пластиной, когда она входит в магнитное поле и выходит из него. В обоих случаях он испытывает силу, противодействующую его движению. Когда он входит слева, поток увеличивается, и поэтому возникает вихревой ток (закон Фарадея) в направлении против часовой стрелки (закон Ленца), как показано. Только правая сторона токовой петли находится в поле, так что слева на нее действует беспрепятственная сила (правило правой руки). Когда металлическая пластина полностью находится внутри поля, вихревой ток отсутствует, если поле однородно, поскольку поток остается постоянным в этой области.Но когда пластина покидает поле справа, поток уменьшается, вызывая вихревой ток по часовой стрелке, который, опять же, испытывает силу слева, еще больше замедляя движение. Аналогичный анализ того, что происходит, когда пластина поворачивается справа налево, показывает, что ее движение также затухает при входе в поле и выходе из него.

Проводящая пластина, проходящая между полюсами магнита : Более подробный взгляд на проводящую пластину, проходящую между полюсами магнита.Когда он входит в поле и выходит из него, изменение потока создает вихревой ток. Магнитная сила на токовой петле препятствует движению. Когда пластина полностью находится внутри однородного поля, нет ни тока, ни магнитного сопротивления.

Когда металлическая пластина с прорезями входит в поле, как показано на, ЭДС индуцируется изменением магнитного потока, но это менее эффективно, поскольку прорези ограничивают размер токовых петель. Более того, в соседних контурах есть токи в противоположных направлениях, и их эффекты нейтрализуются.Когда используется изолирующий материал, вихревые токи чрезвычайно малы, поэтому магнитное затухание на изоляторах незначительно. Если необходимо избегать вихревых токов в проводниках, они могут быть выполнены с прорезями или состоять из тонких слоев проводящего материала, разделенных изоляционными листами.

Вихревые токи, индуцированные в металлической пластине с прорезями : Вихревые токи, индуцируемые в металлической пластине с прорезями, входящие в магнитное поле, образуют небольшие петли, и силы на них имеют тенденцию нейтрализоваться, тем самым делая магнитное сопротивление почти нулевым.

Изменение магнитного потока создает электрическое поле

Закон индукции Фарадея гласит, что изменение магнитного поля создает электрическое поле: [latex] \ varepsilon = — \ frac {\ partial \ Phi_ \ text {B}} {\ partial \ text {t}} [/ latex].

Цели обучения

Опишите взаимосвязь между изменяющимся магнитным полем и электрическим полем

Основные выводы

Ключевые моменты
  • Закон индукции Фарадея — это основной закон электромагнетизма, который предсказывает, как магнитное поле будет взаимодействовать с электрической цепью, создавая электродвижущую силу.
  • Альтернативная дифференциальная форма закона индукции Фарадея выражается в уравнении [latex] \ nabla \ times \ vec {\ text {E}} = — \ frac {\ partial \ vec {\ text {B}}} { \ partial \ text {t}} [/ latex].
  • Закон индукции Фарадея — одно из четырех уравнений Максвелла, управляющих всеми электромагнитными явлениями.
Ключевые термины
  • векторная область : вектор, величина которого соответствует рассматриваемой области и направление которого перпендикулярно плоскости.
  • Уравнения Максвелла : Набор уравнений, описывающих, как электрические и магнитные поля генерируются и изменяются друг другом, а также зарядами и токами.
  • Теорема Стокса : утверждение об интегрировании дифференциальных форм на многообразиях, которое одновременно упрощает и обобщает несколько теорем векторного исчисления.

Мы изучили закон индукции Фарадея в предыдущих атомах. Мы узнали взаимосвязь между наведенной электродвижущей силой (ЭДС) и магнитным потоком.Вкратце, закон гласит, что изменение магнитного поля [латекс] (\ frac {\ text {d} \ Phi_ \ text {B}} {\ text {dt}}) [/ latex] создает электрическое поле [латекс] (\ varepsilon) [/ latex], закон индукции Фарадея выражается как [latex] \ varepsilon = — \ frac {\ partial \ Phi_ \ text {B}} {\ partial \ text {t}} [/ latex], где [латекс] \ varepsilon [/ latex] — это индуцированная ЭДС, а [latex] \ Phi_ \ text {B} [/ latex] — магнитный поток. («N» опущено из нашего предыдущего выражения. Число витков катушки может быть включено в магнитный поток, поэтому коэффициент не является обязательным.) Закон индукции Фарадея — это основной закон электромагнетизма, который предсказывает, как магнитное поле будет взаимодействовать с электрической цепью, создавая электродвижущую силу (ЭДС). В этом Атоме мы узнаем об альтернативном математическом выражении закона.

Эксперимент Фарадея : Эксперимент Фарадея, показывающий индукцию между витками проволоки: жидкая батарея (справа) обеспечивает ток, который течет через небольшую катушку (A), создавая магнитное поле. Когда катушки неподвижны, ток не индуцируется.Но когда малая катушка перемещается внутрь или из большой катушки (B), магнитный поток через большую катушку изменяется, вызывая ток, который регистрируется гальванометром (G).

Дифференциальная форма закона Фарадея

Магнитный поток [латекс] \ Phi_ \ text {B} = \ int_ \ text {S} \ vec {\ text {B}} \ cdot \ text {d} \ vec {\ text {A}} [/ латекс], где [латекс] \ vec {\ text {A}} [/ latex] — это векторная площадь над замкнутой поверхностью S. Устройство, которое может поддерживать разность потенциалов, несмотря на протекание тока, является источником электродвижущей силы. .(EMF) Математически определение [латекс] \ varepsilon = \ oint_ \ text {C} \ vec {\ text {E}} \ cdot \ text {d} \ vec {\ text {s}} [/ latex], где интеграл вычисляется по замкнутому циклу C.

Закон Фарадея теперь можно переписать [latex] \ oint_ \ text {C} \ vec {\ text {E}} \ cdot \ text {d} \ vec {\ text {s}} = — \ frac {\ partial} {\ partial \ text {t}} (\ int \ vec {\ text {B}} \ cdot \ text {d} \ vec {\ text {A}}) [/ latex]. Используя теорему Стокса в векторном исчислении, левая часть равна [latex] \ oint_ \ text {C} \ vec {\ text {E}} \ cdot \ text {d} \ vec {\ text {s}} = \ int_ \ text {S} (\ nabla \ times \ vec {\ text {E}}) \ cdot \ text {d} \ vec {\ text {A}} [/ latex].Также обратите внимание, что в правой части [latex] \ frac {\ partial} {\ partial \ text {t}} (\ int \ vec {\ text {B}} \ cdot \ text {d} \ vec {\ текст {A}}) = \ int \ frac {\ partial \ vec {\ text {B}}} {\ partial \ text {t}} \ cdot \ text {d} \ vec {\ text {A}} [ /латекс]. Таким образом, мы получаем альтернативную форму закона индукции Фарадея: [latex] \ nabla \ times \ vec {\ text {E}} = — \ frac {\ partial \ vec {\ text {B}}} {\ partial \ text {t}} [/ latex]. Это также называют дифференциальной формой закона Фарадея. Это одно из четырех уравнений Максвелла, управляющих всеми электромагнитными явлениями.

Электрогенераторы

Электрические генераторы преобразуют механическую энергию в электрическую; они индуцируют ЭДС, вращая катушку в магнитном поле.

Цели обучения

Объясните, как в электрогенераторах индуцируется электродвижущая сила.

Основные выводы

Ключевые моменты
  • Электрический генератор вращает катушку в магнитном поле, индуцируя ЭДС, заданную как функцию времени величиной ε = NABw sinωt.
  • Генераторы поставляют почти всю мощность для электрических сетей, которые обеспечивают большую часть мировой электроэнергии.
  • Двигатель становится генератором, когда его вал вращается.
Ключевые термины
  • электродвижущая сила : (ЭДС) — напряжение, генерируемое батареей или магнитной силой в соответствии с законом Фарадея. Она измеряется в вольтах, а не в ньютонах, и поэтому на самом деле не является силой.
  • турбина : Любая из различных вращающихся машин, которые используют кинетическую энергию непрерывного потока жидкости (жидкости или газа) для вращения вала.

Электрические генераторы — это устройства, преобразующие механическую энергию в электрическую.Они индуцируют электродвижущую силу (ЭДС), вращая катушку в магнитном поле. Это устройство, преобразующее механическую энергию в электрическую. Генератор заставляет электрический заряд (обычно переносимый электронами) проходить через внешнюю электрическую цепь. Возможные источники механической энергии включают в себя поршневой или турбинный паровой двигатель, воду, падающую через турбину или водяное колесо, двигатель внутреннего сгорания, ветряную турбину, ручной кривошип, сжатый воздух или любой другой источник механической энергии.Генераторы поставляют почти всю мощность для электрических сетей, которые обеспечивают большую часть мировой электроэнергии.

Паровой турбогенератор : Современный паротурбинный генератор.

Базовая настройка

Рассмотрим установку, показанную на. Заряды в проводах петли испытывают магнитную силу, потому что они движутся в магнитном поле. Заряды в вертикальных проводах испытывают силы, параллельные проводу, вызывая токи. Однако те, кто находится в верхнем и нижнем сегментах, ощущают силу, перпендикулярную проводу; эта сила не вызывает тока.Таким образом, мы можем найти наведенную ЭДС, рассматривая только боковые провода. ЭДС движения задается равной ЭДС = Bℓv, где скорость v перпендикулярна магнитному полю B (см. Наш Атом в «ЭДС движения»). Здесь скорость находится под углом θ к B, так что ее составляющая, перпендикулярная B, равна vsinθ.

Схема электрического генератора : Генератор с одной прямоугольной катушкой, вращающейся с постоянной угловой скоростью в однородном магнитном поле, создает ЭДС, синусоидально изменяющуюся во времени.Обратите внимание, что генератор похож на двигатель, за исключением того, что вал вращается для выработки тока, а не наоборот.

Таким образом, в этом случае ЭДС, индуцированная с каждой стороны, равна ЭДС = Bℓvsinθ, и они направлены в одном направлении. Общая ЭДС [латекс] \ varepsilon [/ latex] вокруг петли тогда:

[латекс] \ varepsilon = 2 \ text {Blv} \ sin {\ theta} [/ latex].

Это выражение допустимо, но оно не дает ЭДС как функцию времени. Чтобы найти зависимость ЭДС от времени, предположим, что катушка вращается с постоянной угловой скоростью ω.Угол θ связан с угловой скоростью соотношением θ = ωt, так что:

[латекс] \ varepsilon = 2 \ text {Blv} \ sin {\ omega \ text {t}} [/ latex].

Итак, линейная скорость v связана с угловой скоростью соотношением v = rω. Здесь r = w / 2, так что v = (w / 2) ω, и:

[латекс] \ varepsilon = 2 \ text {Bl} \ frac {\ text {w}} {2} \ omega \ sin {\ omega \ text {t}} = (\ text {lw}) \ text {B } \ omega \ sin {\ omega \ text {t}} [/ латекс].

Учитывая, что площадь петли A = ℓw, и учитывая N петель, мы находим, что:

[латекс] \ varepsilon = \ text {NABw} ~ \ sin {\ omega \ text {t}} [/ latex] — это ЭДС, индуцированная в катушке генератора N витков и площади A, вращающейся с постоянной угловой скоростью в однородное магнитное поле B.

Генераторы, показанные в этом Atom, очень похожи на двигатели, показанные ранее. Это не случайно. Фактически, двигатель становится генератором, когда его вал вращается.

Электродвигатели

Электродвигатель — это устройство, преобразующее электрическую энергию в механическую.

Цели обучения

Объясните, как сила создается в электродвигателях

Основные выводы

Ключевые моменты
  • Большинство электродвигателей используют взаимодействие магнитных полей и токопроводящих проводов для создания силы.
  • Ток в проводнике состоит из движущихся зарядов. Следовательно, катушка с током в магнитном поле также будет ощущать силу Лоренца.
  • В двигателе катушка с током в магнитном поле испытывает силу с обеих сторон катушки, которая создает крутящую силу (называемую крутящим моментом), заставляющую ее вращаться.
Ключевые термины
  • Сила Лоренца : Сила, действующая на заряженную частицу в электромагнитном поле.
  • крутящий момент : вращательное или скручивающее действие силы; (Единица СИ ньютон-метр или Нм; британская единица измерения фут-фунт или фут-фунт)

Основные принципы работы двигателя такие же, как и у генератора, за исключением того, что двигатель преобразует электрическую энергию в механическую энергию (движение).(Сначала прочтите наш атом об электрических генераторах.) Большинство электродвигателей используют взаимодействие магнитных полей и проводников с током для создания силы. Электродвигатели используются в самых разных областях, таких как промышленные вентиляторы, нагнетатели и насосы, станки, бытовая техника, электроинструменты и дисководы.

Лоренц Форс

Если вы поместите движущуюся заряженную частицу в магнитное поле, на нее будет действовать сила, называемая силой Лоренца:

[латекс] \ text {F} = \ text {q} \ times \ text {v} \ times \ text {B} [/ latex]

Правило правой руки : Правило правой руки, показывающее направление силы Лоренца

, где v — скорость движущегося заряда, q — заряд, а B — магнитное поле.Ток в проводнике состоит из движущихся зарядов. Следовательно, катушка с током в магнитном поле также будет ощущать силу Лоренца. Для неподвижного прямолинейного токоведущего провода сила Лоренца составляет:

[латекс] \ text {F} = \ text {I} \ times \ text {L} \ times \ text {B} [/ latex]

где F — сила (в ньютонах, Н), I — ток в проводе (в амперах, А), L — длина провода, находящегося в магнитном поле (в м). , B — напряженность магнитного поля (в теслах, Тл).Направление силы Лоренца перпендикулярно как направлению потока тока, так и магнитного поля, и его можно найти с помощью правила правой руки, показанного на рисунке. Используя правую руку, направьте большой палец в направлении тока, и укажите указательным пальцем в направлении магнитного поля. Ваш третий палец теперь будет указывать в направлении силы.

Крутящий момент : Сила на противоположных сторонах катушки будет в противоположных направлениях, потому что заряды движутся в противоположных направлениях.Это означает, что катушка будет вращаться.

Механика двигателя

И двигатели, и генераторы можно объяснить с помощью катушки, вращающейся в магнитном поле. В генераторе катушка подключена к внешней цепи, которая затем включается. Это приводит к изменению потока, который индуцирует электромагнитное поле. В двигателе катушка с током в магнитном поле испытывает силу с обеих сторон катушки, которая создает крутящую силу (называемую крутящим моментом), заставляющую ее вращаться.Любая катушка, по которой проходит ток, может чувствовать силу в магнитном поле. Эта сила является силой Лоренца, действующей на движущиеся заряды в проводнике. Сила на противоположных сторонах катушки будет в противоположных направлениях, потому что заряды движутся в противоположных направлениях. Это означает, что катушка будет вращаться.

Индуктивность

Индуктивность — это свойство устройства, которое показывает, насколько эффективно оно индуцирует ЭДС в другом устройстве или на самом себе.

Цели обучения

Описание свойств катушки индуктивности с указанием взаимной индуктивности и самоиндукции

Основные выводы

Ключевые моменты
  • Взаимная индуктивность — это влияние двух устройств, индуцирующих друг в друге ЭДС.Изменение тока ΔI 1 / Δt в одном вызывает ЭДС ЭДС2 в секунду: ЭДС 2 = -M ΔI 1 / Δt, где M определяется как взаимная индуктивность между двумя устройствами.
  • Самоиндукция — это эффект, который устройство вызывает само по себе.
  • Устройство, которое демонстрирует значительную самоиндукцию, называется индуктором, и ЭДС, индуцированная в нем изменением тока через него, равна ЭДС = −L ΔI / Δt.
Ключевые термины
  • Закон индукции Фарадея : основной закон электромагнетизма, который предсказывает, как магнитное поле будет взаимодействовать с электрической цепью, создавая электродвижущую силу (ЭДС).
  • трансформатор : статическое устройство, которое передает электрическую энергию от одной цепи к другой с помощью магнитной связи. Их основное назначение — передача энергии между различными уровнями напряжения, что позволяет выбирать наиболее подходящее напряжение для выработки, передачи и распределения электроэнергии по отдельности.

Индукция — это процесс, при котором ЭДС индуцируется изменением магнитного потока. Трансформаторы, например, спроектированы так, чтобы быть особенно эффективными для создания желаемого напряжения и тока с очень небольшими потерями энергии в другие формы (см. Наш Atom в разделе «Трансформаторы.«) Есть ли полезная физическая величина, связанная с тем, насколько« эффективно »данное устройство? Ответ — да, и эта физическая величина называется индуктивностью.

Взаимная индуктивность

Взаимная индуктивность — это влияние закона индукции Фарадея для одного устройства на другое, например, первичная катушка, при передаче энергии вторичной обмотке в трансформаторе. Посмотрите, где простые катушки наводят друг на друга ЭДС.

Взаимная индуктивность катушек : Эти катушки могут вызывать ЭДС друг в друге, как неэффективный трансформатор.Их взаимная индуктивность M указывает на эффективность связи между ними. Здесь видно, что изменение тока в катушке 1 вызывает ЭДС в катушке 2. (Обратите внимание, что «E2 индуцированная» представляет наведенную ЭДС в катушке 2.)

Во многих случаях, когда геометрия устройств фиксирована, магнитный поток изменяется за счет изменения тока. Поэтому мы концентрируемся на скорости изменения тока ΔI / Δt как на причине индукции. Изменение тока I 1 в одном устройстве, катушка 1, индуцирует ЭДС 2 в другом.Мы выражаем это в форме уравнения как

[латекс] \ text {EMF} _2 = — \ text {M} \ frac {\ Delta \ text {I} _1} {\ Delta \ text {t}} [/ latex],

, где M определяется как взаимная индуктивность между двумя устройствами. Знак минус является выражением закона Ленца. Чем больше взаимная индуктивность M, тем эффективнее связь.

Природа здесь симметрична. Если мы изменим ток I2 в катушке 2, мы индуцируем ЭДС 1 в катушке 1, которая равна

[латекс] \ text {EMF} _1 = — \ text {M} \ frac {\ Delta \ text {I} _2} {\ Delta \ text {t}} [/ latex],

, где M то же, что и для обратного процесса.Трансформаторы работают в обратном направлении с такой же эффективностью или взаимной индуктивностью M.

Собственная индуктивность

Самоиндукция, действие закона индукции Фарадея устройства на самого себя, также существует. Когда, например, увеличивается ток через катушку, магнитное поле и магнитный поток также увеличиваются, вызывая противоэдс, как того требует закон Ленца. И наоборот, если ток уменьшается, индуцируется ЭДС, которая препятствует уменьшению. Большинство устройств имеют фиксированную геометрию, поэтому изменение магнитного потока полностью связано с изменением тока ΔI через устройство.Индуцированная ЭДС связана с физической геометрией устройства и скоростью изменения тока. Выдается

[латекс] \ text {EMF} = — \ text {L} \ frac {\ Delta \ text {I}} {\ Delta \ text {t}} [/ latex],

где L — самоиндукция устройства. Устройство, которое демонстрирует значительную самоиндукцию, называется индуктором. Опять же, знак минус является выражением закона Ленца, указывающего на то, что ЭДС препятствует изменению тока.

Количественная интерпретация ЭДС движения

A ЭДС движения — это электродвижущая сила (ЭДС), индуцированная движением относительно магнитного поля B.

Цели обучения

Сформулируйте две точки зрения, которые применяются для расчета электродвижущей силы

Основные выводы

Ключевые моменты
  • Движущая и индуцированная ЭДС — одно и то же явление, наблюдаемое только в разных системах отсчета. Эквивалентность этих двух явлений подтолкнула Эйнштейна к работе над специальной теорией относительности.
  • ЭДС, возникающая из-за относительного движения петли и магнита, задается как [latex] \ varepsilon _ {\ text {motion}} = \ text {vB} \ times \ text {L} [/ latex] (Eq.1), где L — длина объекта, движущегося со скоростью v относительно магнита.
  • ЭДС можно рассчитать с двух разных точек зрения: 1) с точки зрения магнитной силы, действующей на движущиеся электроны в магнитном поле, и 2) с точки зрения скорости изменения магнитного потока. Оба дают одинаковый результат.
Ключевые термины
  • специальная теория относительности : теория, которая (игнорируя эффекты гравитации) согласовывает принцип относительности с наблюдением, что скорость света постоянна во всех системах отсчета.
  • магнитное поле : Состояние в пространстве вокруг магнита или электрического тока, в котором существует обнаруживаемая магнитная сила и где присутствуют два магнитных полюса.
  • рамка отсчета : система координат или набор осей, в пределах которых можно измерить положение, ориентацию и другие свойства объектов в ней.

Электродвижущая сила (ЭДС), индуцированная движением относительно магнитного поля B, называется ЭДС движения. Вы могли заметить, что ЭДС движения очень похожа на ЭДС, вызванную изменением магнитного поля.В этом атоме мы видим, что это действительно одно и то же явление, показанное в разных системах отсчета.

ЭДС движения

В случае, когда проводящая петля перемещается в магнит, показанный на (а), магнитная сила, действующая на движущийся заряд в петле, определяется выражением [латекс] evB [/ латекс] (сила Лоренца, e: заряд электрона).

Петля проводника, движущаяся в магнит : (а) ЭДС движения. Токовая петля переходит в неподвижный магнит. Направление магнитного поля внутрь экрана.(б) Индуцированная ЭДС. Токовая петля неподвижна, а магнит движется.

Из-за силы электроны будут продолжать накапливаться на одной стороне (нижний конец на рисунке), пока на стержне не установится достаточное электрическое поле, препятствующее движению электронов, которое составляет [латекс] \ text {eE} [/ латекс]. Приравнивая две силы, получаем [латекс] \ text {E} = \ text {vB} [/ latex].

Следовательно, двигательная ЭДС на длине L стороны петли определяется как [latex] \ varepsilon _ {\ text {motion}} = \ text {vB} \ times \ text {L} [/ latex] (Eq .1), где L — длина объекта, движущегося со скоростью v относительно магнита.

Индуцированная ЭДС

Поскольку скорость изменения магнитного потока, проходящего через петлю, равна [latex] \ text {B} \ frac {\ text {dA}} {\ text {dt}} [/ latex] (A: площадь петли что магнитное поле проходит), индуцированная ЭДС [латекс] \ varepsilon _ {\ text {индуцированный}} = \ text {BLv} [/ latex] (уравнение 2).

Эквивалентность движущей и индуцированной ЭДС

Из уравнения. 1 и уравнение. 2 мы можем подтвердить, что двигательная и индуцированная ЭДС дают одинаковый результат.Фактически, эквивалентность двух явлений побудила Альберта Эйнштейна исследовать специальную теорию относительности. В своей основополагающей статье по специальной теории относительности, опубликованной в 1905 году, Эйнштейн начинает с упоминания эквивалентности двух явлений:

«…… например, взаимное электродинамическое действие магнита и проводника. Наблюдаемое явление здесь зависит только от относительного движения проводника и магнита, в то время как обычный взгляд проводит резкое различие между двумя случаями, когда одно или другое из этих тел находится в движении.Ведь если магнит находится в движении, а проводник находится в состоянии покоя, в окрестности магнита возникает электрическое поле с определенной энергией , производящее ток в местах, где части проводника находятся расположенный. Но если магнит неподвижен, а проводник движется, электрическое поле поблизости от магнита не возникает. В проводнике, однако, мы находим электродвижущую силу, которой сама по себе не соответствует энергия, но которая вызывает — при условии равенства относительного движения в двух рассмотренных случаях — электрические токи того же пути и силы, что и создаваемые электрическими силами в первом случае.«

Механические работы и электроэнергия

Механическая работа, совершаемая внешней силой для создания ЭДС движения, преобразуется в тепловую энергию; энергия сохраняется в процессе.

Цели обучения

Применить закон сохранения энергии для описания производственной двигательной электродвижущей силы с механической работой

Основные выводы

Ключевые моменты
  • ЭДС движения, создаваемая движущимся проводником в однородном поле, имеет следующий вид [latex] \ varepsilon = \ text {Blv} [/ latex].
  • Чтобы стержень двигался с постоянной скоростью v, мы должны постоянно прикладывать внешнюю силу F ext к стержню во время его движения.
  • Закон Ленца гарантирует, что движение стержня противоположно, и, следовательно, закон сохранения энергии не нарушается.
Ключевые термины
  • ЭДС движения : ЭДС (электродвижущая сила), индуцированная движением относительно магнитного поля.
  • Закон индукции Фарадея : Основной закон электромагнетизма, который предсказывает, как магнитное поле будет взаимодействовать с электрической цепью, создавая электродвижущую силу (ЭДС).

Мы узнали о двигательной ЭДС ранее (см. Наш Атом в «Двигательной ЭДС»). Для простой схемы, показанной ниже, ЭДС движения [латекс] (\ varepsilon) [/ латекс], создаваемая движущимся проводником (в однородном поле), задается следующим образом:

[латекс] \ varepsilon = \ text {Blv} [/ латекс]

, где B — магнитное поле, l — длина проводящего стержня, а v — (постоянная) скорость его движения. ( B , l и v все перпендикулярны друг другу, как показано на изображении ниже.)

ЭДС движения : (a) ЭДС движения = Bℓv индуцируется между рельсами, когда этот стержень перемещается вправо в однородном магнитном поле. Магнитное поле B направлено внутрь страницы, перпендикулярно движущемуся стержню и рельсам и, следовательно, к области, окружающей их. (б) Закон Ленца дает направление индуцированного поля и тока, а также полярность наведенной ЭДС. Поскольку поток увеличивается, индуцированное поле направлено в противоположном направлении или за пределы страницы. Правило правой руки дает указанное направление тока, и полярность стержня будет управлять таким током.

Сохранение энергии

В этом атоме мы рассмотрим систему с точки зрения энергии . Поскольку стержень движется и пропускает ток и , он ощущает силу Лоренца

.

[латекс] \ text {F} _ \ text {L} = \ text {iBL} [/ latex].

Чтобы стержень двигался с постоянной скоростью v , мы должны постоянно прикладывать внешнюю силу F ext (равную величине F L и противоположную по направлению) к стержню во время его движения. .Поскольку стержень движется со скоростью v , мощность P , передаваемая внешней силой, будет:

[латекс] \ text {P} = \ text {F} _ {\ text {ext}} \ text {v} = (\ text {iBL}) \ times \ text {v} = \ text {i} \ варепсилон [/ латекс].

На последнем этапе мы использовали первое уравнение, о котором мы говорили. Обратите внимание, что это в точности мощность, рассеиваемая в контуре (= ток [латекс] \ умноженное на [/ латекс] напряжение). Таким образом, мы заключаем, что механическая работа, совершаемая внешней силой, чтобы стержень двигался с постоянной скоростью, преобразуется в тепловую энергию в контуре.В более общем смысле, механическая работа, совершаемая внешней силой для создания ЭДС движения, преобразуется в тепловую энергию. Энергия сохраняется в процессе.

Закон Ленца

Из «Закона индукции Фарадея и закона Ленца» мы узнали, что закон Ленца является проявлением сохранения энергии. Как мы видим в примере с этим атомом, закон Ленца гарантирует, что движение стержня противодействует из-за склонности природы противодействовать изменению магнитного поля. Если бы наведенная ЭДС была в том же направлении, что и изменение потока, возникла бы положительная обратная связь, заставляющая стержень улетать от малейшего возмущения.

Энергия в магнитном поле

Магнитное поле накапливает энергию. Плотность энергии задается как [латекс] \ text {u} = \ frac {\ mathbf {\ text {B}} \ cdot \ mathbf {\ text {B}}} {2 \ mu} [/ latex].

Цели обучения

Выразите плотность энергии магнитного поля в форме уравнения

Основные выводы

Ключевые моменты
  • Энергия необходима для создания магнитного поля как для работы против электрического поля, создаваемого изменяющимся магнитным полем, так и для изменения намагниченности любого материала в магнитном поле.2 [/ латекс].
Ключевые термины
  • проницаемость : Количественная мера степени намагничивания материала в присутствии приложенного магнитного поля (измеряется в ньютонах на квадратный ампер в единицах СИ).
  • индуктор : Пассивное устройство, которое вводит индуктивность в электрическую цепь.
  • ферромагнетик : Материалы, обладающие постоянными магнитными свойствами.

Энергия необходима для создания магнитного поля как для работы против электрического поля, создаваемого изменяющимся магнитным полем, так и для изменения намагниченности любого материала в магнитном поле.Для недисперсионных материалов эта же энергия высвобождается при разрушении магнитного поля. Следовательно, эту энергию можно смоделировать как «хранящуюся» в магнитном поле.

Магнитное поле, создаваемое соленоидом : Магнитное поле, создаваемое соленоидом (вид в разрезе), описанное с использованием силовых линий. Энергия «хранится» в магнитном поле.

Энергия, запасенная в магнитном поле

Для линейных недисперсных материалов (таких, что B = мкм H, где мкм, называемая проницаемостью, не зависит от частоты), плотность энергии составляет:

[латекс] \ text {u} = \ frac {\ mathbf {\ text {B}} \ cdot \ mathbf {\ text {B}}} {2 \ mu} = \ frac {\ mu \ mathbf {\ text {H}} \ cdot \ mathbf {\ text {H}}} {2} [/ latex].

Плотность энергии — это количество энергии, хранящейся в данной системе или области пространства на единицу объема. Если поблизости нет магнитных материалов, μ можно заменить на μ 0 . Однако приведенное выше уравнение нельзя использовать для нелинейных материалов; необходимо использовать более общее выражение (приведенное ниже).

В общем, дополнительная работа на единицу объема δW , необходимая для того, чтобы вызвать небольшое изменение магнитного поля δ B, составляет:

[латекс] \ delta \ text {W} = \ mathbf {\ text {H}} \ cdot \ delta \ mathbf {\ text {B}} [/ latex].

Когда связь между H и B известна, это уравнение используется для определения работы, необходимой для достижения заданного магнитного состояния. Для гистерезисных материалов, таких как ферромагнетики и сверхпроводники, необходимая работа также зависит от того, как создается магнитное поле. Однако для линейных недисперсионных материалов общее уравнение приводит непосредственно к более простому уравнению плотности энергии, приведенному выше.

Энергия, запасенная в поле соленоида

Энергия, запасенная индуктором, равна количеству работы, необходимой для установления тока через индуктор и, следовательно, магнитного поля.2 [/ латекс].

Трансформаторы

Трансформаторы преобразуют напряжения из одного значения в другое; его функция определяется уравнением трансформатора.

Цели обучения

Примените уравнение трансформатора для сравнения вторичного и первичного напряжений

Основные выводы

Ключевые моменты
  • Трансформаторы часто используются в нескольких точках систем распределения электроэнергии, а также во многих бытовых адаптерах питания.
  • Уравнение трансформатора
  • гласит, что отношение вторичного напряжения к первичному в трансформаторе равно отношению количества витков в их катушках: [латекс] \ frac {\ text {V} _ \ text {s}} {\ text { V} _ \ text {p}} = \ frac {\ text {N} _ \ text {s}} {\ text {N} _ \ text {p}} [/ latex].
  • Если предположить, что сопротивление незначительно, выходная электрическая мощность трансформатора равна его входной. Это приводит нас к другому полезному вопросу: [latex] \ frac {\ text {I} _ \ text {s}} {\ text {I} _ \ text {p}} = \ frac {\ text {N} _ \ текст {p}} {\ text {N} _ \ text {s}} [/ latex]. Если напряжение увеличивается, ток уменьшается. И наоборот, если напряжение уменьшается, ток увеличивается.
Ключевые термины
  • магнитный поток : Мера силы магнитного поля в заданной области.
  • Закон индукции Фарадея : Основной закон электромагнетизма, который предсказывает, как магнитное поле будет взаимодействовать с электрической цепью, создавая электродвижущую силу (ЭДС).

Трансформаторы изменяют напряжение с одного значения на другое. Например, такие устройства, как сотовые телефоны, ноутбуки, видеоигры, электроинструменты и небольшая бытовая техника, имеют трансформатор (встроенный в их съемный блок), который преобразует 120 В в напряжение, соответствующее устройству.Трансформаторы также используются в нескольких точках в системах распределения электроэнергии, как показано на рисунке. Мощность передается на большие расстояния при высоком напряжении, поскольку для данного количества мощности требуется меньший ток (это означает меньшие потери в линии). Поскольку высокое напряжение представляет большую опасность, трансформаторы используются для получения более низкого напряжения в месте нахождения пользователя.

Настройка трансформатора : Трансформаторы изменяют напряжение в нескольких точках в системе распределения электроэнергии. Электроэнергия обычно вырабатывается при напряжении более 10 кВ и передается на большие расстояния при напряжениях более 200 кВ, иногда даже 700 кВ, для ограничения потерь энергии.Распределение электроэнергии по районам или промышленным предприятиям осуществляется через подстанцию ​​и передается на короткие расстояния с напряжением от 5 до 13 кВ. Оно снижено до 120, 240 или 480 В для безопасности на месте отдельного пользователя.

Тип трансформатора, рассматриваемого здесь, основан на законе индукции Фарадея и очень похож по конструкции на устройство, которое Фарадей использовал для демонстрации того, что магнитные поля могут создавать токи (показано на рисунке). Две катушки называются первичной и вторичной катушками.При нормальном использовании входное напряжение подается на первичную обмотку, а вторичная обмотка создает преобразованное выходное напряжение. Мало того, что железный сердечник улавливает магнитное поле, создаваемое первичной катушкой, его намагниченность увеличивает напряженность поля. Поскольку входное напряжение переменного тока, изменяющийся во времени магнитный поток направляется во вторичную обмотку, вызывая ее выходное переменное напряжение.

Простой трансформатор : Типичная конструкция простого трансформатора состоит из двух катушек, намотанных на ферромагнитный сердечник, ламинированный для минимизации вихревых токов.Магнитное поле, создаваемое первичной обмоткой, в основном ограничивается и увеличивается сердечником, который передает его вторичной обмотке. Любое изменение тока в первичной обмотке вызывает ток во вторичной. На рисунке показан простой трансформатор с двумя катушками, намотанными с обеих сторон многослойного ферромагнитного сердечника. Набор катушек на левой стороне сердечника обозначен как первичный, и его номер указан как N p. Напряжение на первичной обмотке равно V p. Набор катушек на правой стороне сердечника обозначен как вторичный, и его номер представлен как N s.Напряжение на вторичной обмотке равно В с. Символ трансформатора также показан под диаграммой. Он состоит из двух катушек индуктивности, разделенных двумя равными параллельными линиями, представляющими сердечник.

Уравнение трансформатора

Для простого трансформатора, показанного на, выходное напряжение V s почти полностью зависит от входного напряжения V p и соотношения количества витков в первичной и вторичной катушках. Закон индукции Фарадея для вторичной обмотки дает ее индуцированное выходное напряжение V с как:

[латекс] \ text {V} _ \ text {s} = — \ text {N} _ \ text {s} \ frac {\ Delta \ Phi} {\ Delta \ text {t}} [/ latex],

, где N s — количество витков вторичной катушки, а Δ / Δt — скорость изменения магнитного потока.Обратите внимание, что выходное напряжение равно индуцированной ЭДС (В с = ЭДС с ), при условии, что сопротивление катушки невелико. Площадь поперечного сечения катушек одинакова с обеих сторон, как и напряженность магнитного поля, поэтому / Δt одинаково с обеих сторон. Входное первичное напряжение V p также связано с изменением магнитного потока:

[латекс] \ text {V} _ \ text {p} = — \ text {N} _ \ text {p} \ frac {\ Delta \ Phi} {\ Delta \ text {t}} [/ latex].

Соотношение этих двух последних уравнений дает полезное соотношение:

[латекс] \ frac {\ text {V} _ \ text {s}} {\ text {V} _ \ text {p}} = \ frac {\ text {N} _ \ text {s}} {\ текст {N} _ \ text {p}} [/ latex].

Это известно как уравнение трансформатора , которое просто утверждает, что отношение вторичного напряжения к первичному в трансформаторе равно отношению количества контуров в их катушках. Выходное напряжение трансформатора может быть меньше, больше или равно входному напряжению, в зависимости от соотношения количества витков в их катушках. Некоторые трансформаторы даже обеспечивают переменный выход, позволяя выполнять подключение в разных точках вторичной обмотки.Повышающий трансформатор — это трансформатор, который увеличивает напряжение, тогда как понижающий трансформатор снижает напряжение.

Если предположить, что сопротивление незначительно, выходная электрическая мощность трансформатора равна его входной. Уравнивание входной и выходной мощности,

[латекс] \ text {P} _ \ text {p} = \ text {I} _ \ text {p} \ text {V} _ \ text {p} = \ text {I} _ \ text {s} \ text {V} _ \ text {s} = \ text {P} _ \ text {s} [/ latex].

Комбинируя эти результаты с уравнением трансформатора, находим:

[латекс] \ frac {\ text {I} _ \ text {s}} {\ text {I} _ \ text {p}} = \ frac {\ text {N} _ \ text {p}} {\ текст {N} _ \ text {s}} [/ latex].

Значит, если напряжение увеличивается, ток уменьшается. И наоборот, если напряжение уменьшается, ток увеличивается.

электрических цепей — Что произойдет с индуктором, если накопленная энергия не найдет пути для разряда?

Предположим, что индуктор подключен к источнику, а затем источник отключен. 2 $.

При размыкании переключателя вы, очевидно, внезапно прерываете ток $ I $. Дифференциальное уравнение между напряжением $ V_L $ и током $ I $ через катушку индуктивности имеет вид $$ V_L = L \ frac {dI} {dt} $$ или для конечного временного шага $$ V_L = L \ frac {\ Delta I} {\ Delta t}. $$ Теперь в нашем случае $ I $ меняется с $ \ frac {V_0} {R} $ на $ 0 $, и, следовательно, $ \ Delta I = — \ frac {V_0} {R} $. А для идеального свитча это $ \ Delta t = 0 $. Итак, мы ожидаем, что индуктор вырабатывает напряжение $$ V_L = L \ frac {\ Delta I} {\ Delta t} = — L \ frac {V_0 / R} {0} = — \ infty.$$ Это может быть правильно? Ну, почти.

Когда напряжение на размыкающем переключателе достигает нескольких 1000 вольт, воздух между контактами переключателя ионизируется и становится электрическим проводником. В соответствии с « Электрический пробой — Газы » Воздух начинает пробиваться при 3000 В / мм. Вы действительно увидите и услышите искру в переключателе.


(изображение из электронных схем сборки — Что такое индуктор ?, немного изменен мной)

Что будет в этом случае с запасенной энергией, током и напряжением индуктора?

В течение нескольких миллисекунд ток продолжает течь через уже разомкнутый переключатель, прохождение через ионизированный воздух искры.Энергия, запасенная в катушке индуктивности, рассеивается в этой искре.

Резюме: Катушка индуктивности не «хочет» прерывания тока и поэтому индуцирует достаточно высокое напряжение, чтобы ток продолжался.

Примечание: во многих приложениях в области электротехники индукционная искра такого типа — крайне нежелательная особенность. Этого можно избежать, добавив в схему обратный диод. Однако в некоторых приложениях (например, при электрическом зажигании в бензиновых двигателях) требуется индукционная искра.

.

Добавить комментарий

Ваш адрес email не будет опубликован. Обязательные поля помечены *